Sie sind auf Seite 1von 93

ENADE COMENTADO 2008

Fsica

Chanceler
Dom Dadeus Grings
Reitor
Joaquim Clotet
Vice-Reitor
Evilzio Teixeira
Conselho Editorial
Ana Maria Lisboa de Mello
Elaine Turk Faria
rico Joo Hammes
Gilberto Keller de Andrade
Helenita Rosa Franco
Jane Rita Caetano da Silveira
Jernimo Carlos Santos Braga
Jorge Campos da Costa
Jorge Luis Nicolas Audy Presidente
Jos Antnio Poli de Figueiredo
Jurandir Malerba
Lauro Kopper Filho
Luciano Klckner
Maria Lcia Tiellet Nunes
Marlia Costa Morosini
Marlise Arajo dos Santos
Renato Tetelbom Stein
Ren Ernaini Gertz
Ruth Maria Chitt Gauer
EDIPUCRS
Jernimo Carlos Santos Braga Diretor
Jorge Campos da Costa Editor-chefe
Maria Eullia Pinto Tarrag
Dlcio Basso
(Organizadores)












ENADE COMENTADO 2008
Fsica












Porto Alegre
2011
Dados Internacionais de Catalogao na Publicao (CIP)
Ficha Catalogrfca elaborada pelo Setor de Tratamento da Informao da BC-PUCRS.
EDIPUCRS Editora Universitria da PUCRS
Av. Ipiranga, 6681 Prdio 33
Caixa Postal 1429 CEP 90619-900
Porto Alegre RS Brasil
Fone/fax: (51) 3320 3711
e-mail: edipucrs@pucrs.br - www.pucrs.br/edipucrs
EDIPUCRS, 2011
CAPA Rodrigo Valls
REVISO DE TEXTO Rafael Saraiva
EDITORAO ELETRNICA Gabriela Viale Pereira
Questes retiradas da prova do ENADE 2008 da Fsica
E56 ENADE comentado 2008 : fsica [recurso eletrnico] /
organizadores, Maria Eullia Pinto Tarrag, Dlcio
Basso. Dados eletrnicos. Porto Alegre :
EDIPUCRS, 2011.
92 p.
Sistema requerido: Adobe Acrobat Reader
Modo de Acesso: <http://www.pucrs.br/edipucrs/>
ISBN 978-85-397-0083-7 (on-line)
1. Ensino Superior Brasil Avaliao. 2. Exame
Nacional de Desempenho de Estudantes. 3. Fsica
Ensino Superior. I. Tarrag, Maria Eullia Pinto. II. Basso,
Dlcio.
CDD 378.81
SUMRIO
APRESENTAO ..................................................................................................... 8
Ana Maria Marques da Silva
COMPONENTE ESPECFICO - NCLEO COMUM
QUESTO 11 ........................................................................................................... 12
Artur Majolo Scheid, Maria Eullia Pinto Tarrag e Dlcio Basso
QUESTO 12 ........................................................................................................... 13
Dlcio Basso e Maria Eullia Pinto Tarrag
QUESTO 13 ........................................................................................................... 15
Artur Majolo Scheid, Maria Eullia Pinto Tarrag e Dlcio Basso
QUESTO 14 ........................................................................................................... 17
Maria Eullia Pinto Tarrag e Dlcio Basso
QUESTO 15 ........................................................................................................... 18
Maria Eullia Pinto Tarrag e Dlcio Basso
QUESTO 16 ........................................................................................................... 19
Rafael L. Zimmer
QUESTO 17 ........................................................................................................... 22
Dlcio Basso e Maria Eullia Pinto Tarrag
QUESTO 18 ........................................................................................................... 23
Natthan Ruschel Soares, Maria Eullia Pinto Tarrag e Dlcio Basso
QUESTO 19 ........................................................................................................... 25
Dlcio Basso
QUESTO 20 ........................................................................................................... 27
Elias Cantarelli Hoffmann, Maria Eullia Pinto Tarrag e Dlcio Basso
QUESTO 21 ........................................................................................................... 28
Elias Cantarelli Hoffmann, Maria Eullia Pinto Tarrag e Dlcio Basso
QUESTO 22 ........................................................................................................... 30
Artur Majolo Scheid, Maria Eullia Pinto Tarrag e Dlcio Basso
QUESTO 23 ........................................................................................................... 32
Elaine Evani Streck
QUESTO 24 ........................................................................................................... 34
Natthan Ruschel Soares, Maria Eullia Tarrag e Dlcio Basso
QUESTO 25 ........................................................................................................... 35
Juliane Bernardes Marcolino, Maria Eullia Tarrag e Dlcio Basso
QUESTO 26 ........................................................................................................... 37
Elaine Evani Streck e Janana Galho Borges
QUESTO 27 ........................................................................................................... 39
Aldoir Rigoni
QUESTO 28 ........................................................................................................... 40
Dlcio Basso
QUESTO 29 ........................................................................................................... 42
Elaine Evani Streck e Janana Galho Borges
QUESTO 30 - DISCURSIVA .................................................................................. 44
Alexandre Ferret, Maiara Oliveira Dalenogare, Mrcio Galhardi, Maria Eullia
Pinto Tarrag e Dlcio Basso
COMPONENTE ESPECFICO - BACHARELADO
QUESTO 31 ........................................................................................................... 47
Maria do Carmo Baptista Lagreca e Ricardo Meurer Papalo
QUESTO 32 ........................................................................................................... 50
Elias Cantarelli Hoffmann, Maria Eullia Pinto Tarrag e Dlcio Basso
QUESTO 33 ........................................................................................................... 52
Maria Eullia Pinto Tarrag e Dlcio Basso
QUESTO 34 - QUESTO ANULADA ................................................................... 54
Adriano Moehlecke, Izete Zanesco e Aline Cristiane Pan
QUESTO 35 ........................................................................................................... 56
Cssio Stein Moura
QUESTO 36 ........................................................................................................... 58
Dlcio Basso e Maria Eullia Pinto Tarrag
QUESTO 37 ........................................................................................................... 60
Maria Eullia Pinto Tarrag e Dlcio Basso
QUESTO 38 ........................................................................................................... 62
Cssio Stein Moura
QUESTO 39 - DISCURSIVA .................................................................................. 64
Sayonara Salvador Cabral da Costa
QUESTO 40 - DISCURSIVA .................................................................................. 66
Cssio Stein Moura
COMPONENTE ESPECFICO - LICENCIATURA
QUESTO 41 ........................................................................................................... 71
Joo Bernardes da Rocha Filho
QUESTO 42 ........................................................................................................... 73
Joo Bernardes da Rocha Filho
QUESTO 43 ........................................................................................................... 75
Joo Bernardes da Rocha Filho
QUESTO 44 ........................................................................................................... 77
Joo Bernardes da Rocha Filho
QUESTO 45 ........................................................................................................... 79
Joo Bernardes da Rocha Filho
QUESTO 46 - QUESTO ANULADA ................................................................... 81
Aldoir Rigoni
QUESTO 47 ........................................................................................................... 83
Aldoir Rigoni
QUESTO 48 ........................................................................................................... 84
Aldoir Rigoni
QUESTO 49 - DISCURSIVA .................................................................................. 86
Maria Eullia Pinto Tarrag e Dlcio Basso
QUESTO 50 - DISCURSIVA .................................................................................. 88
Adriana Schier e Sayonara Cabral da Costa
LISTA DE CONTRIBUINTES ................................................................................... 91

8 Maria Eullia Tarrag e Dlcio Basso (Orgs.)
APRESENTAO
No ensino superior, as avaliaes institucionais possibilitam a anlise de
diversas dimenses do ensino, da pesquisa e da extenso, evidenciando objetivos e
compromissos das instituies e de seus cursos de graduao.
O Sistema Nacional de Avaliao da Educao Superior (Sinaes), institudo
atravs da Lei n
o
10.861, de 14/04/2004, tem como objetivo assegurar o processo
continuado de avaliao das instituies de educao superior, dos cursos de
graduao e do desempenho dos estudantes.
Como forma de atingir as instituies de ensino superior em sua totalidade, o
Sinaes possui trs componentes principais: a avaliao interna e externa das
instituies, a avaliao dos cursos e a avaliao do desempenho dos estudantes,
por meio do Exame Nacional de Desempenho dos Estudantes (ENADE).
O ENADE constitudo por um questionrio socioeconmico e uma prova. Por
meio do questionrio socioeconmico, possvel compor o perfil dos estudantes,
integrando informaes do seu contexto s suas percepes sobre a instituio de
ensino e suas vivncias.
A prova busca avaliar como o estudante capaz de utilizar suas competncias e
habilidades, assim como analisar sua evoluo, por meio da avaliao em dois
momentos: no primeiro e no ltimo ano da graduao. Alm das competncias
profissionais, a formao geral e a abordagem dos temas transversais so avaliados,
enriquecendo o sistema de avaliao e agregando-lhe elementos de reflexo.
As questes da prova, de natureza objetiva e discursiva, priorizam temas
contextualizados e atuais, problematizados na forma de situaes problemas e
estudos de caso, propiciando respostas por meio de mltipla escolha.
A prova compe-se de duas partes: Formao Geral e Componente Especfico.
A parte da Formao Geral comum s provas aplicadas a todos os cursos
participantes do ENADE daquela edio. Tem como objetivo investigar
competncias, habilidades e conhecimentos gerais dos estudantes, analisando a
compreenso de temas relacionados realidade nacional e internacional, que no
pertencem necessariamente ao mbito especfico da profisso.
ENADE Comentado 2008: Fsica 9
A parte do Componente Especfico contempla as especificidades de cada
curso, tanto no domnio dos conhecimentos quanto nas habilidades esperadas para
cada perfil profissional. Tem como objetivo investigar os contedos do curso, por
meio de nveis distintos de habilidades e saberes.
Para o curso de Fsica, o Componente Especfico da prova do ENADE 2008 foi
dividido em duas partes, sendo uma parte denominada Ncleo Comum (questes 11
a 30), realizada por todos os estudantes de Fsica, outra parte para avaliao do
componente especfico do Bacharelado (questes 31 a 40) e outra para o
componente especfico da Licenciatura (questes 41 a 50).
Na parte de avaliao do Ncleo Comum foram apresentadas 20 (vinte)
questes que se referiam aos contedos gerais da formao em Fsica, sendo 1
(uma) questo discursiva e 19 (dezenove) questes de mltipla escolha. Para a
avaliao do componente especfico do Bacharelado em Fsica havia 2 (duas)
questes discursivas e 8 (oito) questes objetivas. Da mesma forma, o componente
especfico da Licenciatura em Fsica foi avaliado por meio de 2 (duas) questes
discursivas e 8 (oito) questes de mltipla escolha.
Esta publicao d continuidade a Srie ENADE Comentado, publicada pela
EDIPUCRS, que apresenta a resoluo comentada das provas de diferentes cursos,
desde as edies de 2004 do ENADE. O formato eletrnico da coleo permite que
os interessados tenham acesso universal e facilitado s publicaes.
O ENADE Comentado 2008: Fsica, ora apresentado, disponvel gratuitamente
na Internet, apresenta a resoluo comentada de todas as 40 (quarenta) questes do
componente especfico das provas aplicadas aos estudantes do curso de Fsica, tanto
em relao aos contedos gerais (Ncleo Comum), quanto aos componentes
especficos do Bacharelado e da Licenciatura. Na publicao esto includos,
inclusive, os comentrios das questes que foram posteriormente anuladas pelo MEC.
Esta publicao o resultado concreto de um processo de reflexo realizado
por professores e estudantes do Curso de Fsica da PUCRS, a partir da realizao
do ENADE 2008. Para comemorar os bons resultados obtidos pelos alunos do Curso
de Fsica da PUCRS, no final de 2009, foi organizado um evento, no qual estudantes
concluintes e professores foram convidados a apresentarem suas percepes sobre
a prova e comentarem algumas questes. Nesta ocasio, na qual estiveram
10 Maria Eullia Tarrag e Dlcio Basso (Orgs.)
presentes professores da Faculdade de Fsica e estudantes do curso, foi realizado
um convite a todos que desejassem colaborar na publicao das questes
comentadas do ENADE Fsica. Os estudantes, entusiasmados com a possibilidade
de uma produo conjunta, procuraram a orientao de professores para a
elaborao da resoluo comentada de questes de sua escolha. Estamos certos de
que esta experincia produziu um impacto positivo na formao e amadurecimento
dos estudantes que aceitaram tal desafio, assim como uma aproximao e
comprometimento de professores e estudantes em relao importncia desta
avaliao.
Alm da materializao da profcua parceria entre docentes e discentes da
Faculdade de Fsica, esta publicao pretende representar um subsdio de estudo e
consulta para estudantes e professores, possibilitando a discusso e resoluo de
problemas em diferentes tpicos de Fsica.
A organizao desta publicao foi realizada pelos professores Dlcio Basso e
Maria Eullia Pinto Tarrag, que assumiram esse desafio e souberam motivar o
grupo para a produo conjunta, realizando, alm da resoluo de algumas
questes, a reviso e uniformizao de todo o material.
O trabalho cooperativo para a publicao do ENADE Comentado 2008: Fsica
envolveu um total de 13 professores da Faculdade de Fsica e 9 estudantes e
diplomados do Curso de Fsica da PUCRS.
Aos organizadores e a todos os colaboradores desta publicao, agradecemos
pela dedicao e competncia com que realizaram este trabalho aqui apresentado.

Porto Alegre, maio de 2011
Ana Maria Marques da Silva
Diretora da Faculdade de Fsica/PUCRS
























COMPONENTE ESPECFICO
NCLEO COMUM

12 Maria Eullia Pinto Tarrag e Dlcio Basso (Orgs.)
QUESTO 11
No dia 19 de agosto de 2008 foi lanado, pelo foguete russo Proton Breeze M o I4-
F3, um dos maiores satlites j construdos, que ser utilizado para servios de
telefonia e Internet. O conjunto foguete + satlite partiu de uma posio vertical.
Sendo a massa m do satlite igual a 6 toneladas, a massa M do foguete igual a 690
toneladas e a velocidade de escape dos gases no foguete (v
gases
) igual a 1.500 m/s,
qual a quantidade mnima de gs expelida por segundo (m
gases
/t) para que o
foguete eleve o conjunto no instante do lanamento?
(Considere g = 10 m/s
2
)
(A) 9,3 x 10
3
kg/s
(B) 4,6 x 10
3
kg/s
(C) 2,3 x 10
3
kg/s
(D) 2,3 x 10
2
kg/s
(E) 2,2 x 10
4
kg/s

Gabarito: B
Tipo de questo: Escolha simples, com indicao da alternativa correta.
Resolutores: Artur Majolo Scheid, Prof. Dr. Maria Eullia Pinto Tarrag e Prof. Me.
Dlcio Basso
Comentrio:
Para resolver essa questo vamos lembrar o Princpio de Conservao da
Quantidade de Movimento de acordo com o surgimento de foras internas em um
sistema no altera a quantidade de movimento do mesmo.
Nessa questo o nosso sistema constitudo pelo foguete com o satlite mais
os gases que esto sendo expelidos. Portanto, a fora mnima exercida pelos gases
expelidos deve ser igual, em mdulo, ao peso do foguete com o satlite.
Logo,
gases
gases
v
t
m
g m M

= + ) ( . Isolando
t
m
gases

e substituindo as grandezas
pelos seus valores numricos obtemos
gases
gases
v
g m M
t
m
) ( +
=

=
s
m
s
m
kg
1500
10 ) 10 6 10 690 (
2
3 3
+
s
kg 3
10 6 , 4 = . Portanto, a
resposta certa a alternativa (B).

ENADE Comentado 2008: Fsica 13
QUESTO 12
A figura abaixo representa o movimento de uma bola, em um plano vertical,
registrado com uma fonte de luz pulsada a 20 Hz.
(As escalas vertical e horizontal so iguais.)


Supondo que a acelerao da gravidade local seja igual a 10 m/s
2
, qual o mdulo
da componente horizontal da velocidade da bola?
(A) 2 m/s.
(B) 3 m/s
(C) 4 m/s
(D) 5 m/s
(E) 6 m/s

Gabarito: A
Tipo de questo: Escolha associada com indicao da resposta correta.
Resolutores: Prof. Me. Dlcio Basso e Prof. Dr. Maria Eullia Pinto Tarrag
Comentrio:
O mdulo da velocidade horizontal da bola constante e igual a
t
x
v
x

= ,
portanto precisamos descobrir a distncia x percorrida num dado intervalo de tempo t.

14 Maria Eullia Pinto Tarrag e Dlcio Basso (Orgs.)
A anlise da figura permite escrever que a bola leva o tempo
s s t 5 , 0 10
20
1
= = para descer desde o ponto mais alto de sua trajetria, em que sua
velocidade na direo vertical
y
v nula, at a ltima representao da bola,
percorrendo uma distncia na vertical aproximadamente igual a 12 = y unidades.
Por outro lado, a relao cinemtica nos diz que
. 25 , 1 ) 5 , 0 ( /
2
10
2
12
2 2 2
m s s m t
g
unidades y = = = = Portanto, cada unidade equivale
a 0,104m. Sabemos que enquanto a bola cai, ela percorre 10 unidades na direo
horizontal, portanto, o mdulo da velocidade horizontal da bola ser
s m
s
m
t
x
v
x
/ 2
5 , 0
104 , 0 10
=

= , sendo a opo correta a alternativa (A).



ENADE Comentado 2008: Fsica 15
QUESTO 13
Uma brincadeira de criana que mora perto de um riacho atravess-lo usando uma
corda amarrada a uma rvore perto da margem. Dependendo da resistncia da
corda, essa travessia pode no se
concretizar. Para avaliar o perigo da
travessia, pode-se usar como modelo o
movimento do pndulo, e calcular a tenso
mxima que a corda pode suportar.
Considerando que a corda faz, inicialmente,
um ngulo de 60com a vertical, qual a
tenso mxima a ser suportada pela corda
para que uma criana de 30 kg atravesse o
riacho?
(Considere g = 10 m/s
2
)
(A) 200 N
(B) 300 N
(C) 600 N
(D) 900 N
(E) 1.200 N

Gabarito: C
Tipo de questo: Escolha simples com indicao da resposta correta.
Resolutores: Artur Majolo Scheid, Prof. Dr. Maria Eullia Pinto Tarrag e Prof. Me.
Dlcio Basso
Comentrio:
O menino pendurado na corda est submetido a uma fora de tenso e a sua
fora peso. A tenso mxima T
max
ocorre no ponto mais baixo da trajetria, em que a
fora peso mg, para baixo, e a tenso, para cima, esto na mesma direo. Por
outro lado, como a trajetria descrita pelo menino um arco de circunferncia, a
fora resultante sobre o menino, no ponto mais baixo da sua trajetria, a fora
centrpeta dada por F
c
=T
max
mg.
T
I
P
L
E
R
,

P
.
A
.
;

M
O
S
C
A
,

G
.

F

s
i
c
a

p
a
r
a

c
i
e
n
t
i
s
t
a
s

e

e
n
g
e
n
h
e
i
r
o
s
,

V
1

-

M
e
c

n
i
c
a
,

O
s
c
i
l
a

e
s
,

O
n
d
a
s
,

T
e
r
m
o
d
i
n

m
i
c
a
.

R
i
o

d
e

J
a
n
e
i
r
o
:

L
T
C
,

2
0
0
6
.


16 Maria Eullia Pinto Tarrag e Dlcio Basso (Orgs.)
Isolando T
max
e lembrando que
r
mv
ma F
c c
2
= = onde m a massa do menino, v
a sua velocidade no ponto mais baixo da trajetria e r raio da trajetria (igual ao
comprimento da corda), teremos mg
r
mv
T + =
2
max
.
Precisamos, ento, descobrir quanto vale v
2
.
Para tanto, vamos considerar a conservao da
energia mecnica que nos permite escrever a
igualdade
2
2
mv
mgh = ou gh v 2
2
= . A altura h
obtida da relao trigonomtrica mostrada na figura
ao lado, em que
2
) 5 , 0 1 (
r
h = ;

60 cos r r h = .
Portanto gr v =
2
.
Substituindo a relao anterior na expresso
de T
max
obtemos:
N mg mg
r
mgr
T
mx
600 2 = = + =
.
Portanto, a alternativa correta a (C).

ENADE Comentado 2008: Fsica 17
QUESTO 14
Um disco gira livremente, com velocidade angular , em torno de um eixo vertical
que passa pelo seu centro. O momento de inrcia do disco em relao ao eixo I
1
.
Um segundo disco, inicialmente sem rotao, colocado no mesmo eixo e cai sobre
o primeiro disco, como mostra a figura. Aps algum tempo, o atrito faz com que os
dois discos girem juntos. Se o momento de inrcia do segundo disco I
2
, qual a
velocidade angular final de rotao do conjunto?
(A)
(B)
(C)
(D)
(E)


Gabarito: D
Tipo de questo: Escolha simples com indicao da resposta correta.
Resolutores: Prof.Dr. Maria Eullia Pinto Tarrag e Prof. Me. Dlcio Basso
Comentrio:
O sistema formado pelos dois discos, um j em rotao e pelo outro, sem
rotao, que cai sobre o primeiro. Assim, na ausncia de torques externos, deve
ocorrer a conservao do momento angular:
1
I L
i
= =
f f
I I L ) (
2 1
+ = .
Ento,
2 1
1
I I
I
f
+
= . Portanto, a alternativa correta a (D).

18 Maria Eullia Pinto Tarrag e Dlcio Basso (Orgs.)
QUESTO 15
Uma jovem me prepara o banho para o seu beb. Ela sabe que a temperatura da
gua da torneira de 20 C, e que a temperatura ideal da gua para o banho de
36 C. Quantos litros de gua fervendo a me deve misturar com a gua da torneira
para obter 10 litros de gua na temperatura ideal para o banho?
(A) 2,5
(B) 2,0
(C) 1,5
(D) 1,0
(E) 0,5

Gabarito: B
Tipo de questo: Escolha simples com indicao da resposta correta.
Resolutores: Prof.Dr. Maria Eullia Pinto Tarrag e Prof. Me. Dlcio Basso
Comentrio:
Para resolver essa questo devemos lembrar que a soma do calor ganho pela
gua da torneira com o calor perdido pela gua fervendo nula, isto , 0 = +
p g
Q Q .
Logo, 0 = +
q q f f
T c m T c m , em que m
f
a massa de gua fria e
f
T a variao de
sua temperatura; semelhantemente m
q
a massa de gua quente e
q
T a variao
de sua temperatura. Lembrando que o calor especfico da gua 1,0cal/gC e que a
temperatura da gua fervendo 100C, a 1atm, ento a equao anterior, fica:
0 ) 100 36 ( ) 20 36 ( = +
q f
m m , resultando em 0 64 16 =
q f
m m (Eq.A).
Por outro lado, sabemos que a soma das massas de gua fervendo (gua
quente) e de gua da torneira (gua fria) 10kg, uma vez que a massa especfica
da gua 1,0kg/litro. Logo, podemos escrever kg m m
f q
10 = + (Eq.B). Substituindo a
equao B na equao A encontramos que a massa da gua quente 2,0kg,
correspondendo a um volume de 2,0litros, o que nos permite marcar como correta a
alternativa (B).

ENADE Comentado 2008: Fsica 19
QUESTO 16
Cinco sensores foram utilizados para medir a temperatura de um determinado corpo.
As curvas de calibrao da resistncia eltrica, em funo da temperatura destes
sensores, so apresentadas no grfico abaixo.




(1) Germnio
(2) Vidro-Carbono
(3) Platina
(4) Cernox
(5) Rox







Analisando-se o grfico, foram feitas as afirmativas a seguir.
I - O sensor (2) s deve ser utilizado para temperaturas superiores a 20 K.
II - Para temperaturas entre 1 K e 3 K apenas o sensor (5) pode ser utilizado.
III - Quando a resistncia do sensor (1) atingir o valor de cerca de 7, o sensor (4)
estar com uma resistncia um pouco superior a 2 k.
IV - O sensor (3) o nico a ser empregado para temperaturas na faixa de 20 K a
300 K.
So verdadeiras APENAS as afirmaes
(A) I e II
(B) I e IV
(C) II e III
(D) II e IV
(E) III e IV


20 Maria Eullia Pinto Tarrag e Dlcio Basso (Orgs.)
Gabarito: C
Tipo de questo: Escolha combinada com indicao da alternativa correta.
Resolutor: Rafael L. Zimmer Aluno Concluinte
Comentrio:
A afirmativa I falsa, pois o sensor (2) pode ser utilizado na faixa de
temperaturas entre 4K at 300K.



A afirmativa II verdadeira, pois Para temperaturas entre 1K e 3K apenas o
sensor (5) pode ser utilizado.




ENADE Comentado 2008: Fsica 21
A afirmativa III verdadeira, pois quando a resistncia do sensor (1) atingir o
valor de cerca de 7, o sensor (4) estar com uma resistncia um pouco superior a
2k.



A afirmativa IV falsa, pois os sensores 2 e 4 podem tambm ser
empregados na faixa de temperaturas entre 20 K e 300 K.



Portanto, a alternativa correta a (C).

22 Maria Eullia Pinto Tarrag e Dlcio Basso (Orgs.)
QUESTO 17
Uma certa quantidade de um gs ideal ocupa um volume inicial V
i
presso p
i
e
temperatura T
i
. O gs se expande at o volume V
f
(V
f
> V
i
), segundo dois processos
distintos: (1) a temperatura constante e (2) adiabaticamente.
Com relao quantidade de calor Q fornecida, ao trabalho W realizado e
variao de energia interna E de cada processo, pode-se afirmar que
I - Q
1
= Q
2

II - Q
1
> Q
2

III - E
1
= E
2

IV - E
1
< E
2

V - W
1
> W
2

So verdadeiras APENAS as afirmaes
(A) I e III
(B) I e IV
(C) II e V
(D) III e V
(E) IV e V

Gabarito: C
Tipo de questo: Escolha combinada com indicao da resposta correta.
Resolutores: Prof. Me. Dlcio Basso e Prof. Dr. Maria Eullia Pinto Tarrag
Comentrio:
A variao da energia interna do gs para o processo isotrmico (1) nula,
0
1
= E , logo Q
1
=W
1
= ) ln(
i
f
i
V
V
nRT , como
f
V >
i
V ento Q
1
=W
1
>0.
A variao da energia interna do gs para o processo adiabtico (2) igual ao
negativo do trabalho,
2 2
W E = , uma vez que o calor envolvido nesse processo
nulo, Q
2
=0. Como W
2
>0, ento
2
E <0.
Como o trabalho W equivalente a rea sob a curva no diagrama PV,
presso em funo de volume, fcil verificar que W
1
>W
2
, pois no processo 2 o gs
esfria na medida que expande, portanto T
2
<T
1
.
Logo, somente as afirmativas II (Q
1
>Q
2
) e V (W
1
>W
2
) so verdadeiras,
correspondendo opo (C).

ENADE Comentado 2008: Fsica 23
QUESTO 18
Em 1816, o escocs Robert Stirling criou uma mquina trmica a ar quente que
podia converter em trabalho boa parte da energia liberada pela combusto externa
de matria-prima. Numa situao idealizada, o ar tratado como um gs ideal com
calor especfico molar C
v
= 5 R/2, onde R a constante universal dos gases. A
mquina idealizada por Stirling representada pelo diagrama P versus V da figura
abaixo. Na etapa C D (isotrmica), a mquina interage com o reservatrio quente,
e na etapa A B (tambm isotrmica), com o reservatrio frio. O calor liberado na
etapa isovolumtrica D A
recuperado integralmente na
etapa B C, tambm
isovolumtrica. So conhecidas
as temperaturas das isotermas
T
1
e T
2
, os volumes V
A
e V
B
e o
nmero de moles n de ar
contido na mquina.

HALLIDAY, D; RESNICK, R; WALKER, J.
Fundamentos de Fsica, v.2, 4 ed. Rio
de Janeiro: LTC, 1996.

Qual o rendimento do ciclo e sua variao total de entropia?
(A)
(B)

(C)
(D)
(E)



24 Maria Eullia Pinto Tarrag e Dlcio Basso (Orgs.)
Gabarito: D
Tipo de questo: Escolha simples com indicao da resposta correta.
Resolutores: Natthan Ruschel Soares, Prof. Dr. Maria Eullia Pinto Tarrag e Prof.
Me. Dlcio Basso
Comentrio:
Para resolver essa questo o estudante deve lembrar que o rendimento
termodinmico de um motor dado por
for
res
Q
W
= , em que
res
W o trabalho resultante
no ciclo e Q
for
o calor fornecido.
O trabalho resultante dado por W
AB
+ W
CD
, uma vez que os trabalhos W
BC
e
W
DA
so nulos, pois corresponde a etapas isomtricas.
Para um processo isotrmico o trabalho realizado = W
i
f
V
V
nRT ln que igual
ao calor trocado.
No enunciado afirma-se que o calor liberado na etapa isovolumtrica de D
para A, Q
DA
, recuperado integralmente na etapa isovolumtrica de B para C, Q
BC
,
assim o calor fornecido no ciclo Q
for
=W
CD
=
C
D
V
V
nRT ln
2
.
Portanto, a eficincia do motor ser
CD
CD AB
for
res
W
W W
Q
W +
= = =
CD
AB
W
W
+ 1 .
Reescrevendo:
) ln(
) ln(
1
2
1
C
D
A
B
V
V
nRT
V
V
nRT
+ = . Porm, ) ( ) (
D
C
A
B
V
V
V
V
= , assim ) ln( ) ln(
C
D
A
B
V
V
V
V
= ,
ento
2
1
1
T
T
= .
Como o ciclo formado por transformaes reversveis, a variao resultante
da entropia nula. Logo, a alternativa correta a (D).

ENADE Comentado 2008: Fsica 25
QUESTO 19
Em fins do sculo XVIII, a Academia de Cincias da Frana publicou o trabalho de
C.A. de Coulomb intitulado Primeira memria sobre a eletricidade e o magnetismo,
no qual foram relatados a construo de uma balana de toro e experimentos
que relacionavam corpos carregados eletricamente com foras a distncia entre
esses corpos.
Posteriormente, M. Faraday concebeu um sistema de linhas invisveis que
existiriam no espao entre as cargas eltricas, contribuindo para o desenvolvimento
do conceito de campo eltrico. Considerando esse contexto, analise as afirmaes a
seguir.

I - Para Coulomb, as interaes eltricas eram foras a distncia entre as cargas.
II - As linhas invisveis de Faraday no correspondem s linhas de fora de um
campo eltrico.
III - O conceito de campo eltrico permitiu a substituio do conceito de ao a
distncia.

Est(o) correta(s) APENAS a(s) afirmativa(s)
(A) I
(B) II
(C) III
(D) I e II
(E) I e III.

Gabarito: E
Tipo de questo: Escolha combinada com indicao da resposta correta.
Resolutor: Prof. Me. Dlcio Basso
Comentrio:
A afirmativa I correta, porque a expresso de Coulomb no leva em conta o
meio entre as cargas, considera apenas a fora que cada carga exerce diretamente
sobre a outra.

26 Maria Eullia Pinto Tarrag e Dlcio Basso (Orgs.)
A afirmativa II incorreta, pois as linhas invisveis de Faraday correspondem
s linhas de fora do campo eltrico.
A afirmativa III correta uma vez que o conceito de campo eltrico substituiu
a ao a distncia pela interao de cada carga com o campo.


ENADE Comentado 2008: Fsica 27
QUESTO 20
Qual das equaes do eletromagnetismo apresentadas a seguir implica a no-
existncia de monoplos magnticos?
(A)
(B)
(C)
(D)
(E)


Gabarito: B
Tipo de questo: Escolha simples com indicao da resposta correta.
Resolutores: Elias Cantarelli Hoffmann, Prof. Dr. Maria Eullia Pinto Tarrag e
Prof. Me. Dlcio Basso
Comentrio:
A lei de Gauss, para o campo magntico, expressa a inseparabilidade dos
polos magnticos e o fato de serem fechadas as linhas de induo magntica,
fazendo com que o fluxo magntico resultante atravs de uma superfcie fechada
seja igual a zero. Em funo disso, de acordo com o teorema da divergncia
aplicado ao vetor induo magntica, pode-se afirmar que a equao da alternativa
(B) implica a no existncia de monopolos magnticos.


28 Maria Eullia Pinto Tarrag e Dlcio Basso (Orgs.)
QUESTO 21
Uma barra metlica puxada de modo a deslocar-se, com velocidade , sobre dois
trilhos paralelos e condutores, separados por uma distncia , como mostra a figura
abaixo.


Um resistor de resistncia eltrica R conecta os dois trilhos, e um campo magntico
uniforme atravessa, perpendicularmente, o plano do conjunto, preenchendo todo o
espao. Qual a intensidade da corrente eltrica que atravessa o resistor?

(A)
(B)
(C)
(D)
(E)


Gabarito: B
Tipo de questo: Escolha simples com indicao da alternativa correta.
Resolutores: Elias Cantarelli Hoffmann, Prof. Dr. Maria Eullia Pinto Tarrag e
Prof. Me. Dlcio Basso
Comentrio:
O fluxo magntico atravs do circuito est variando em funo do aumento da
rea correspondente aos trilhos. Em um intervalo de tempo dt , a barra metlica se
desloca em uma distncia dt v e a rea aumenta de vdt dA = (Figura 1).

ENADE Comentado 2008: Fsica 29
Considerando-se positivo o sentido do vetor rea entrando no plano da pgina,
paralelamente ao vetor B

, tem-se que o fluxo magntico atravs do circuito


positivo e durante o intervalo de tempo dt o mesmo aumentar de acordo com:
. ) 0 cos( vdt B BdA d
B
= =
Dessa forma tem-se que a f.e.m. (fora eletromotriz) induzida dada por
v B
dt
d
B
=

= .
Em consequncia, a corrente eltrica i no circuito ser representada pelo
mdulo da f.e.m. dividido pela resistncia R, de acordo com a seguinte equao
R
v B
i

= , o que corresponde a alternativa (B).


30 Maria Eullia Pinto Tarrag e Dlcio Basso (Orgs.)
QUESTO 22
Uma onda se propaga em uma corda, representada na figura abaixo em dois
momentos sucessivos. O intervalo de tempo entre esses dois momentos de 0,2s.

Com relao propagao dessa onda, foram feitas as afirmativas a seguir.
I - A velocidade da onda 40 cm/s.
II - A freqncia da onda 1,25 Hz.
III - As ondas esto defasadas de .
IV - As ondas esto deslocadas de meio comprimento de onda.

So corretas APENAS as afirmaes
(A) I e II
(B) I e IV
(C) II e III
(D) II e IV
(E) III e IV

Gabarito: C
Tipo de questo: Escolha combinada com indicao da resposta correta
Resolutores: Artur Majolo Scheid, Prof. Dr. Maria Eullia Pinto Tarrag e Prof. Me.
Dlcio Basso
Comentrio:
A figura dada na questo indica a distncia entre duas cristas consecutivas,
logo se sabe que o comprimento de onda () =80,0cm. As duas ondas mostradas

ENADE Comentado 2008: Fsica 31
esto deslocadas por /4, como indicado na figura ao lado, o que equivale dizer que
a distncia percorrida pela onda foi 20,0cm no intervalo de tempo de 0,2s. Portanto,
a velocidade de propagao dessa onda s cm
s
cm
v / 100
2 , 0
0 , 20
= = . Logo, a afirmativa I
falsa.


Por outro lado, a velocidade de propagao de uma onda pode ser dada por
f v = , isso significa que a frequncia f dessa onda vale Hz
cm
s cm v
f 25 , 1
80
/ 100
= = =

.
Portanto, a afirmativa II correta.
A defasagem entre as ondas /4, em termos de distncia percorrida pela
onda, ou
2
4
2

= . Logo, a afirmativa III correta.


A afirmativa IV falsa, pois j comentamos que as ondas esto deslocadas
por /4.
Assim, a alternativa correta a letra (C).

32 Maria Eullia Pinto Tarrag e Dlcio Basso (Orgs.)
QUESTO 23
Em uma experincia de interferncia entre duas fendas iguais, utilizou-se um feixe
de luz monocromtica, de comprimento de onda = 500 nm, incidindo
perpendicularmente ao plano que contm as fendas.



O padro de interferncia observado no anteparo, posicionado a uma distncia
L=1,0 m do plano das fendas, est representado na figura a seguir com a
intensidade I em funo da posio x.


Considerando-se os dados apresentados, qual a distncia d entre as duas fendas?
(A) 1,70 cm
(B) 0,85 cm
(C) 1,50 mm
(D) 0,30 mm
(E) 0,15 mm

Gabarito: D
Tipo de questo: Escolha simples com indicao da alternativa correta.
Resolutor: Prof. Dr. Elaine Evani Streck

ENADE Comentado 2008: Fsica 33
Comentrio:
A situao descrita na questo retrata o clssico experimento de Young que
permitiu modelar a interferncia de dois feixes luminosos coerentes de comprimento
de onda provenientes de duas fendas separadas por uma pequena distncia d
como foi mostrado na figura dada na questo.
Nessas condies, num anteparo localizado a uma distncia L das fendas,
forma-se uma figura de interferncia envolta por uma figura de difrao, tambm
mostrada na questo.
As faixas claras correspondem s posies cuja interferncia construtiva,
sendo sua posio dada por m dsen = , em que m=0, 1, 2,... representa a ordem
do mximo.
A figura fornecida no enunciado permite identificar com relativa clareza que o
terceiro mximo de difrao ocorre a 0,5cm do mximo central.
Levando em conta a distncia de 1m entre as fendas e o anteparo, pode-se
considerar que o ngulo correspondente a esse terceiro mximo pequeno e,
portanto, pode-se considerar que tg de modo que rad ,
cm
cm ,
L
x
005 0
100
5 0
= = .
Pelo mesmo motivo sen e a relao para a posio angular dos mximos na
interferncia de fenda dupla pode ser aproximada por m d = .
Assim sendo, tem-se para d o valor: mm , m
rad ,
m
d 3 0 10 3
005 0
10 500 3
4
9
= =

=

.
Portanto, a alternativa correta a (D).

34 Maria Eullia Pinto Tarrag e Dlcio Basso (Orgs.)
QUESTO 24
Na flauta, o tubo sonoro ressoa notas diferentes, com freqncias diferentes, de
acordo com o nmero de furos fechados pelos dedos do flautista.



Com os furos todos tampados, gerada a nota l, de 440 Hz. Abrindo alguns furos,
de modo a ressoar 2/3 do tubo, a freqncia, em hertz, ser
(A) 145
(B) 293
(C) 660
(D) 880
(E) 1.000

Gabarito: C
Tipo de questo: Escolha simples com indicao da resposta correta.
Resolutores: Natthan Ruschel Soares, Prof. Dr. Maria Eullia Tarrag e Prof. Me.
Dlcio Basso
Comentrio:
Para resolver essa questo o estudante precisa saber que o padro da onda
estacinria gerada dentro da flauta, com todos os furos tampados, apresenta
L =
4
1
, sendo o comprimento de onda do som e L o comprimento da flauta.
Abrindo alguns furos de modo a ressoar apenas 2/3 do tubo, o comprimento de onda
ficar 2/3 menor, e, como consequncia, a frequncia ficar 3/2 maior. Isto ,
Hz Hz 660 440
2
3
= . Assim, a alternativa correta a (C).

ENADE Comentado 2008: Fsica 35
QUESTO 25
Microondas so ondas eletromagnticas que, quando absorvidas pela gua, geram
calor no interior do alimento por aumentar a vibrao de suas molculas. Na porta
de vidro de um forno de microondas existe uma rede metlica de proteo. A rede
metlica tem orifcios de 2 mm de dimetro. Durante a operao, possvel ver o
interior do forno. No entanto, o cozinheiro est protegido da radiao microondas.
A esse respeito, foram feitas as afirmativas a seguir.

I - A radiao com comprimento de onda no infravermelho prximo (~1m)
bloqueada pela grade.
II - A largura dos orifcios da ordem de grandeza do comprimento de onda da luz
visvel.
III - A rede metlica impede a transmisso das microondas, mas no impede a
transmisso da radiao visvel, por causa da diferena entre as freqncias.
IV - As cincias histricas tm especificidades metodolgicas: seus objetos so
transitrios e atravessados por interesses de classes.
V - O comprimento de onda da radiao microondas maior do que o da luz visvel.

Est(o) correta(s) APENAS a(s) afirmao(es)
(A) I
(B) II
(C) III
(D) I e II
(E) III e IV

Gabarito: E
Tipo de questo: Escolha combinada com indicao da resposta correta.
Resolutores: Juliane Bernardes Marcolino, Prof. Dr. Maria Eullia Tarrag e Prof.
Me. Dlcio Basso
Comentrio:
I Afirmativa incorreta, pois a largura dos orifcios da grade de 2mm, o que
equivalente a 2000m. Logo, radiaes com comprimentos menores do que esse
valor no sero bloqueadas pela rede metlica do forno de micro-ondas.

36 Maria Eullia Pinto Tarrag e Dlcio Basso (Orgs.)
II Afirmativa incorreta, j que os comprimentos de onda da luz visvel esto,
aproximadamente, entre 700nm e 400nm, valores muito menores do que 2000m.
III Afirmativa correta, pois os comprimentos de ondas das micro-ondas
esto entre 1m e 1mm. Logo, sero barradas pela grade. J a luz visvel, de
comprimentos de ondas aproximadamente entre 0,0007mm e 0,0004mm no
barrada pela grade.
IV - Afirmativa correta, pelo o que j foi comentado.
Portanto, a alternativa correta a (E).

ENADE Comentado 2008: Fsica 37
QUESTO 26
Sobre o Modelo Atmico de Bhr, so feitas as seguintes afirmaes:
I - o tomo composto de um ncleo e de uma eletrosfera;
II - o momento angular orbital do eltron um mltiplo inteiro de h/2, onde h a
Constante de Planck;
III - a freqncia da radiao eletromagntica emitida pelo tomo varia
continuamente entre os dois valores correspondentes s rbitas de maior e
menor energia.
Para Bhr, verdadeiro SOMENTE o que se afirma em
(A) I
(B) II
(C) III
(D) I e II
(E) II e III

Gabarito: D
Tipo de questo: Escolha combinada com indicao da resposta correta.
Resolutores: Prof. Dr. Elaine Evani Streck e Prof. Dr. Janana Galho Borges
Comentrio:
Quando Bohr sugeriu seu modelo para o tomo, o modelo aceito era o de
Rutherford. Para o modelo de Rutherford os eltrons giravam em torno do ncleo de
forma anloga ao movimento dos planetas em torno do Sol. Tal modelo tinha alguns
problemas no solucionados. Um deles era o fato dos eltrons no colapsarem no
ncleo, uma vez que, de acordo com a teoria eletromagntica uma carga acelerada
(acelerao centrpeta no caso) emite energia na forma de radiao o que faria com
que os mesmos descrevessem rbitas cada vez menores at colapsarem no ncleo.
Outra grande lacuna do modelo era a de que no justificava a emisso de radiao
caracterstica (o espectro de linhas).
Bohr, para criar um modelo que solucionasse esses problemas, sugeriu que o
eltron s poderia orbitar o ncleo em rbitas circulares com energias bem definidas,
ou seja, cada rbita corresponde a um valor de energia. Quanto maior a energia do
eltron mais afastada do ncleo estaria a rbita por ele ocupada.

38 Maria Eullia Pinto Tarrag e Dlcio Basso (Orgs.)
Para mudar de rbita, o eltron precisava absorver, ou emitir energia. A
emisso de energia ocorre na forma de um fton cuja energia igual diferena de
energia entre as rbitas.
final inicial
E E h =
em que a frequncia da radiao e
final
E e
inicial
E so as energias
correspondentes s orbitas final e inicial entre as quais o eltrons realiza a transio.
O momento angular ) (L , sugerido nesse modelo, baseado nas ideias de
Planck e Einstein e dado por:
2
h
n L = , em que h a constante de Planck e n o
nmero quntico principal e sempre assume valores inteiros e positivos ,...) 3 , 2 , 1 ( = n .
Baseado no texto acima, os itens I e II so corretos, enquanto o item III est
errado, uma vez que a frequncia da radiao eletromagntica descontnua.
Assim, a resposta certa a alternativa (D).

ENADE Comentado 2008: Fsica 39
QUESTO 27
A radiao trmica emitida por estrelas pode ser modelada como semelhante de
um corpo negro. A radincia espectral do corpo negro mxima para uma
freqncia ou comprimento de onda. A Lei de Wien estabelece uma relao entre
esse comprimento de onda
mx
e a temperatura absoluta T do objeto, atravs de
uma constante determinada, experimentalmente, como igual a 2,9 x 10
-3
m.K.
Usando a Lei de Wien para a estrela Polar, com
mx
= 350 nm, qual a temperatura
absoluta dessa estrela, em milhares de kelvins?
(A) 1,7
(B) 3,9
(C) 5,7
(D) 8,3
(E) 11,0

Gabarito: D
Tipo de questo: Escolha simples com indicao da alternativa correta.
Resolutor: Prof. Dr. Aldoir Rigoni
Comentrio:
A energia irradiada por um corpo negro relaciona-se com a temperatura T e o
comprimento de onda . Quando a temperatura se eleva, a energia emitida aumenta
e o pico de distribuio desta energia se desloca para comprimentos de ondas
menores. Wien, a partir da experimentao, estabelece que o produto entre a
temperatura absoluta T e o comprimento de onda
mx
, que corresponde ao pico da
distribuio de energia, igual a 2,9 x 10
3
mK, ou seja mK T
3
max
10 9 , 2

= . Usando
este modelo matemtico, para o valor do comprimento de onda dado,
mx
=350nm,
determina-se que a temperatura absoluta dessa estrela, em milhares de kelvins,
8,3; portanto a alternativa correta a letra D.

40 Maria Eullia Pinto Tarrag e Dlcio Basso (Orgs.)
QUESTO 28
Em relao Teoria da Relatividade Restrita, analise as afirmaes a seguir.
I - O mdulo da velocidade da luz no vcuo independente das velocidades do
observador ou da fonte.
II - A Teoria Eletromagntica de Maxwell compatvel com a Teoria da Relatividade
Restrita.
III - As leis da Fsica so as mesmas em todos os referenciais inerciais.

Est correto o que se afirma em
(A) I, apenas.
(B) II, apenas.
(C) III, apenas.
(D) I e III, apenas.
(E) I, II e III.

Gabarito: E
Tipo de questo: Escolha combinada com indicao da resposta correta.
Resolutor: Prof. Me. Dlcio Basso
Comentrio:
I Correta.
Vamos analisar a afirmativa II (A Teoria Eletromagntica de Maxwell
compatvel com a Teoria da Relatividade Restrita):
Os relativistas, FitzGerald, Larmor e Lorentz, alteraram o significado de
algumas grandezas nas equaes do eletromagnetismo Clssico ou de Maxwell.
Em particular o significado de v

na equao B v q F

= , que, originalmente, era uma


velocidade em relao ao meio onde se propagam as partculas e ondas
eletromagnticas, chamado de ter eletromagntico (que j havia includo o ter
luminfero) e passou a ser interpretado como velocidade em relao a um
observador inercial.
Essa alterao conceitual introduziu assimetrias no Eletromagnetismo
Clssico, tornando diversas equaes no invariantes nas mudanas de referenciais

ENADE Comentado 2008: Fsica 41
inerciais. Essas assimetrias foram eliminadas com o emprego das equaes de
transformao de coordenadas relativsticas. Portanto, a Teoria Eletromagntica
Clssica, originalmente, no era compatvel com a Teoria da Relatividade Restrita,
mas foi tornada compatvel
Na percepo do resolutor, essa afirmativa no correta; no entanto, muitos
textos de Fsica no destacam o aspecto de que a Teoria Eletromagntica Clssica
foi adaptada para ficar compatvel com a Teoria da Relatividade Restrita, o que
induziria a considerar essa afirmativa como correta.
.
III- Correta.
Assim, para o resolutor, a alternativa correta a (D); no entanto, o gabarito d
como alternativa correta a (E).

42 Maria Eullia Pinto Tarrag e Dlcio Basso (Orgs.)
QUESTO 29
Do ponto de vista da Fsica Moderna, a respeito do espectro de energias do
oscilador harmnico, so feitas as seguintes afirmaes:
I - o espectro de energia contnuo;
II - o espectro de energia discreto;
III - em acordo com o Princpio da Correspondncia de Bohr e para grandes
nmeros qnticos a separao de energias entre dois nveis consecutivos
torna-se desprezvel quando comparada com estas energias.
Est(o) correta(s) APENAS a(s) afirmao(es)
(A) I
(B) II
(C) III
(D) I e II
(E) II e III

Gabarito: E
Tipo de questo: Escolha combinada com indicao da resposta correta.
Resolutores: Prof. Dr. Elaine Evani Streck e Prof. Dr. Janana Galho Borges
Comentrio:
O oscilador harmnico pode ser usado para entendermos melhor as vibraes
moleculares.
A energia potencial de um oscilador harmnico simples, a partir da equao
de Schroedinger independente do tempo, resulta em um conjunto de estados de
energia discretos dados por: ,
2
1
|
.
|

\
|
+ = n
m
k
E para ,... 2 , 1 , 0 = n , em que k a
constante da fora de restaurao, ma massa e a constante de Planck dividida por
2. Assim, dessa equao podemos inferir que o item I errado e o II correto.
No Princpio da Correspondncia, Bohr afirma que para grandes nmeros
qunticos a mecnica quntica se reduz a mecnica clssica, criando uma relao
entre as duas teorias. O Princpio da Correspondncia pode ser ilustrado, com maior
clareza, se analisarmos os nveis de energia no tomo de hidrognio, por exemplo,

ENADE Comentado 2008: Fsica 43
em que
2
6 . 3
n
eV
E
n
= . A diferena de energia entre os nveis um e dois de 10,2eV,
enquanto entre os nveis nove dez ela igual a 0,032eV.
Assim, a afirmao do item III correta, pois para n muito grande j estamos
no limite da clssica.
Portanto, a resposta correta a letra (E).

44 Maria Eullia Pinto Tarrag e Dlcio Basso (Orgs.)
QUESTO 30 - DISCURSIVA
Numa competio entre estudantes de Fsica de vrias instituies, um grupo
projeta uma mquina trmica hipottica que opera entre somente dois reservatrios
de calor, a temperaturas de 250 K e 400 K. Nesse projeto, a mquina hipottica
produziria, por ciclo, 75 J de trabalho, absorveria 150 J de calor da fonte quente e
cederia 75 J de calor para a fonte fria.

a) Verifique se essa mquina hipottica obedece ou no Primeira Lei da
Termodinmica, justificando a sua resposta.
(valor: 3,0 pontos)

b) Verifique se essa mquina hipottica obedece ou no Segunda Lei da
Termodinmica, justificando a sua resposta.
(valor: 3,0 pontos)

c) Considerando que o menor valor de entropia 0,1 J/K, e que o trabalho realizado
por ciclo 75 J, esboce um diagrama Temperatura versus Entropia para um
Ciclo de Carnot que opere entre esses dois reservatrios de calor, indicando os
valores de temperaturas e entropias.
(valor: 4,0 pontos)

Tipo de questo: Discursiva
Resolutores: Alexandre Ferret, Maiara Oliveira Dalenogare, Mrcio Galhardi, Prof.
Dr. Maria Eullia Pinto Tarrag e Prof. Me. Dlcio Basso
Comentrio:
a) A Primeira Lei da Termodinmica diz que a energia conservada segundo
a equao: W Q U = . Como a mquina recebeu 150J de calor, transforma 75J
em trabalho e perde 75J para a fonte fria, a energia se conserva, obedecendo a
primeira lei.


ENADE Comentado 2008: Fsica 45
b) A Segunda Lei da Termodinmica permite concluir que o rendimento
termodinmico limite de um motor
q
f
T
T
e =1 , que, nesse caso, resulta em
K
K
e
400
250
1 = =37,5%. No entanto, pelo enunciado, a mquina hipottica teria um
rendimento de
J
J
e
150
75
= =50,0%. Portanto, a mquina hipottica no obedece a
Segunda Lei da Termodinmica.

c) A variao da entropia para um processo isotrmico
T
Q
S = ou
T S Q . = . Assim, o trabalho no ciclo S T S T Q Q W
f q f q
= = . Substituindo pelos
valores numricos fornecidos tem-se:
S K S K J = 250 400 75 ; 5 , 0
150
75
= =
K
J
S J/K.

Logo, o valor superior da entropia (0,1+0,5)J/K=0,6J/K.


























COMPONENTE ESPECFICO
BACHARELADO


ENADE Comentado 2008: Fsica 47
QUESTO 31
Uma dada molcula orgnica, em determinada diluio, apresenta o espectro de
absorvncia descrito pela figura abaixo.



Supondo que esta molcula possa ser tratada como uma estrutura linear em que
quatro eltrons estejam aprisionados em um poo quntico infinito, qual o valor
estimado de L?

(A) 0,6 nm
(A) 1nm
(B) 0,6 m
(C) 1 m
(D) 2 m

Gabarito: B
Tipo de questo: Mltipla Escolha
Resolutores: Prof. Me. Maria do Carmo Baptista Lagreca e Prof. Dr. Ricardo
Meurer Papalo

48 Maria Eullia Pinto Tarrag e Dlcio Basso (Orgs.)
Comentrio:
O modelo supe quatro eltrons aprisionados num poo de potencial infinito
de largura L, em que
(1)
Pelo princpio de excluso de Pauli, nunca pode haver mais de um eltron
ocupando o mesmo estado quntico. Como temos 4 eltrons num poo de potencial
infinito, 2 eltrons estaro no estado fundamental (n=1) e 2 eltrons estaro no
primeiro estado excitado (n=2). Considerando que os eltrons so partculas
idnticas, os eltrons tm a energia correspondente energia do nvel em que se
encontram. Assim, os 2 eltrons do nvel 1 tem a mesma energia E
1
e os 2 eltrons
do nvel 2 tem a mesma energia E
2
.
Para analisarmos o grfico da absorvncia (ou absorbncia), em funo do
comprimento de onda, precisamos ter em mente que a absorvncia, dada por
, a capacidade intrnseca dos materiais em absorver radiaes em
frequncia especfica. Em que ) ( I a intensidade da luz transmitida e ) (
0
I a
intensidade da luz incidente no comprimento de onda especfico .
O espectro de absorvncia mostra claramente dois picos (um em e
outro em ) que indicam duas transies eletrnicas permitidas.
Pela relao , pode-se perceber que o maior comprimento de onda de
absoro, indica a transio eletrnica de menor energia: a de um eltron do estado
n=2, para o primeiro estado desocupado n=3:
(2)
Portanto, basta considerar esse primeiro pico de absoro em ~565 nm para
determinar L a partir das equaes (1) e (2). Os demais picos de absoro tambm
podem ser utilizados para calcular L e devem dar resultado similar. Inicialmente
vamos calcular L a partir da transio n=2 n=3.
Igualando as equaes (1) e (2) para a transio considerada temos:
, usando e


ENADE Comentado 2008: Fsica 49







Assim, o valor estimado de L 1nm, correspondente a letra (B).

50 Maria Eullia Pinto Tarrag e Dlcio Basso (Orgs.)
QUESTO 32
Num dia de chuva, uma nuvem eletricamente carregada pode se descarregar
produzindo relmpagos. Uma nuvem tpica se encontra a uma altura de 5.000 m do
solo, com uma diferena de potencial de 10 milhes de volts em relao ao solo. Em
um laboratrio, uma estudante de Fsica realiza uma experincia para medir a
rigidez dieltrica do ar seco usando um capacitor de placas planas e paralelas cuja
distncia entre as placas pode ser variada. Mantendo uma diferena de potencial
constante entre as placas e iguais a 24 kV, a estudante diminui lentamente a
distncia entre elas at que, na distncia de 0,8 cm, observa uma centelha no ar
entre as placas.
Quais so os valores do campo eltrico entre a nuvem e o solo e da rigidez dieltrica
do ar seco, respectivamente?
(A) 2,0 kV/m e 3,0 x 10
6
V/m
(B) 2,0 kV/m e 1,9 x 10
4
V/m
(C) 5,0 kV/m e 3,0 x 10
6
V/m
(D) 10 kV/m e 3,0 x 10
5
V/m
(E) 20 kV/m e 1,9 x 10
6
V/m

Gabarito: A
Tipo de questo: Escolha simples com indicao da resposta correta.
Resolutores: Elias Cantarelli Hoffmann, Prof. Dr. Maria Eullia Pinto Tarrag e
Prof. Me. Dlcio Basso
Comentrio:
Para se encontrar o valor da rigidez dieltrica do ar seco, deve-se calcular o
valor do mdulo do campo eltrico mximo, entre as placas planas do capacitor,
para que no ocorra ruptura dieltrica. Considerando-se que a diferena de potencial
entre as placas do capacitor constante, e que na distncia de 0,80 cm entre as
placas tem-se a intensidade mxima do campo eltrico, antes da ruptura, o valor da
rigidez dieltrica :
m V
m
V
cm
kV
x
V
E
placas
placas
mx
/ 10 0 , 3
10 0 , 8
10 24
80 , 0
24
6
3
3
=

= =

.

ENADE Comentado 2008: Fsica 51
Semelhantemente, o valor do campo eltrico entre a nuvem e o solo :
3
6
10 0 , 2
5000
10 10
=

=
m
V
x
V
E
nuvem
solo
nuvem
solo

V/m=2,0kV/m.
Assim, correta a alternativa (A).

52 Maria Eullia Pinto Tarrag e Dlcio Basso (Orgs.)
QUESTO 33
Quando uma onda eletromagntica plana penetra em um meio material, a sua
amplitude decai com a distncia de penetrao, ou seja, ela tem a sua amplitude
atenuada pelo meio. A profundidade de penetrao da onda a profundidade na
qual a intensidade do campo foi reduzida a aproximadamente 1/3 do valor inicial.
Define-se a profundidade da penetrao como:

onde
a condutividade do meio;
a permeabilidade magntica do meio;
a freqncia angular da onda.
Para onda com freqncia especfica
0
, a condutividade na prata
prata
= 3x10
-7

(m)
-1
e no mar
mar
= 4,0 x 10
-7
(m)
-1
, e para ambos = 4 x 10
-7
N/A
2
.
A esse respeito, analise as afirmaes a seguir.
I - A penetrao da onda maior na prata do que no mar.
II - Para um meio condutor com condutividade constante, uma onda com menor
comprimento de onda tem uma profundidade de penetrao maior do que outra
onda com maior comprimento de onda.
III - A uma profundidade de 2 da superfcie, a sua amplitude ser aproximadamente
10% da amplitude original.

Esto corretas SOMENTE as afirmaes
(A) I
(B) II
(C) III
(D) I e III
(E) II e III

Gabarito: D
Tipo de questo: Escolha combinada com indicao da resposta correta.
Resolutores: Prof. Dr. Maria Eullia Pinto Tarrag e Prof. Me. Dlcio Basso

ENADE Comentado 2008: Fsica 53
Comentrio:
Essa uma questo de interpretao de texto, pois todas as informaes
esto dadas no enunciado. Portanto, passaremos diretamente a anlise das
afirmativas.
A profundidade de penetrao da onda maior na prata do que no mar, pois
inversamente proporcional raiz da condutividade eltrica , e
prata
<
mar
;
portanto, a afirmativa I est CORRETA.
A Velocidade angular pode ser dada por


v
f 2 2 = = , sendo o
comprimento de onda. Ento, para um meio condutor com condutividade constante a
profundidade de penetrao da onda diretamente proporcional raiz de ;
portanto, a afirmativa II INCORRETA.
A profundidade de penetrao da onda a profundidade na qual a
intensidade do campo, ou seja, a amplitude da onda, 1/3 do valor inicial. Logo,
quando a profundidade de penetrao for 2 a amplitude ser 10 , 0
9
1
3
1
3
1
= ;
portanto, a afirmativa III est CORRETA.
Assim, a letra correta a (D).



QUESTO 34 (QUESTO ANULADA)

O enxofre cristalino, um slido transparente de cor amarelo plido por absorver
apenas a luz azul e nenhuma outra cor, um isolante eltrico. Qual o gap de
energia do enxofre cristalino?
(Considere hc = 1,2 eV.m)
(A) 2,5
(B) 4,5
(C) 5,0
(D) 5,5
(E) 6,0

Gabarito: A
Tipo de questo: Escolha simples com indicao da resposta correta.
Resolutores: Prof. Dr. Adriano Moehlecke, Prof. Dr. Izete Zanesco e Prof. Dr.
Aline Cristiane Pan

Comentrio:
Um cristal s pode absorver os ftons de luz que apresentam energia
equivalente a seu gap de energia. Por outro lado, sabe-se que a energia de um
fton de luz dada por

hc
E = , em que h a constante de Planck, c a velocidade
da luz no vcuo e o comprimento de onda da luz. Portanto, para resolver essa
questo devemos lembrar que o comprimento de onda correspondente ao azul
0,48m e, consequentemente,
m
m eV
E

48 , 0
2 , 1
= = 2,5eV. No entanto, caso o aluno no
54 Maria Eullia Pinto Tarrag e Dlcio Basso (Orgs.)

ENADE Comentado 2008: Fsica 55
se lembre do correspondente ao azul, mesmo que utilizasse qualquer outro
na faixa do espectro visvel, encontraria que a alternativa mais prxima do valor
que obteve a (A).
Logo, a resposta certa a letra (A). Observa-se que na questo fornecida,
faltou explicitar a unidade de energia para as alternativas a serem escolhidas.

56 Maria Eullia Pinto Tarrag e Dlcio Basso (Orgs.)
QUESTO 35
Um metal unidimensional tem um eltron de conduo por tomo a temperatura
T=0K. O espaamento interatmico no metal D. Supondo que os eltrons movem-
se livremente, qual a energia de Fermi E
F
?
( = h / 2 a Constante de Planck e m a massa do eltron)
(A)
(B)
(C)
(D)

(E)


Gabarito: A
Tipo de questo: Mltipla Escolha
Resolutor: Prof. Dr. Cssio Stein Moura
Comentrio:
Nesse problema, podemos aproximar cada tomo do cristal como um poo de
potencial quadrado infinito. Por ser um metal, consideramos que o eltron de
conduo move-se livremente no interior do poo de potencial e, portanto, a
equao de Schrdinger unidimensional independente do tempo para o problema :

E
x m
=

2
2 2
2

.

ENADE Comentado 2008: Fsica 57
Considerando-se as condies de contorno:

= =
= =
0 ) (
0 ) 0 (
D x
x

, esta equao
diferencial apresenta como soluo a autofuno: ) sin( ) ( kx A x = , em que k
definido pelo autovalor de energia
m
k
E
2
2 2

= .
Para que as condies de contorno sejam obedecidas, ou seja, a funo de
onda se anule nas paredes necessrio que:
D
n
k

= . Logo,
2
2
2 2
2
n
mD
E
n

= , n =
1, 2, 3 ...
Pelo princpio de excluso de Pauli, em se tratando de frmions, cada nvel
energtico (n =1, 2, 3 ... ) somente pode ser ocupado por no mximo dois eltrons.
Se considerarmos que o metal est sujeito a temperatura nula e que hajam N
eltrons no poo de potencial, todos os nveis at
2
N
n = esto ocupados.
Defini-se como energia de Fermi (E
F
) a energia do ltimo nvel ocupado, ou
seja, o nvel em que
2
N
n = . Assim,
2
2
2 2
2 /
2 2
|
.
|

\
|
= =
=
N
mD
E E
N n F

. Como, do enunciado,
N= 1, chegamos expresso para a energia de Fermi:
2
2 2
2
2
2 2
8 2
1
2 mD mD
E
F

=
|
.
|

\
|
= .
Assim, a alternativa correta a (A).



QUESTO 36
O LHC (Large Hadron Collider), acelerador de partculas que entrou em operao
este ano, busca uma nova Fsica na escala de at 14 TeV. A principal busca pela
partcula chamada Higgs, que supostamente gera as massas das partculas
responsveis pela interao nuclear fraca, como o W
+
e o W

. Essas partculas so
muito massivas se comparadas a outras como o prton e o eltron. Suas massas de
repouso so da ordem de 82 GeV. Elas sero geradas em quantidade no LHC e
com energias que podem chegar, em um experimento tpico, a 500 GeV para o W
+

ou o W

. Essas partculas so muito instveis, pois decaem rapidamente. Estima-se


que suas vidas mdias sejam de 3 x 10
25
s, em seu referencial de repouso. No
referencial do laboratrio (LHC), qual seria sua vida mdia, num experimento tpico?
(Dados: 1 TeV = 10
3
GeV = 10
12
eV
1eV = 1,6 x 10
19
J)
(A) 9 x 10
25
s
(B) 18 x 10
25
s
(C) 27 x 10
25
s
(D) 3 x 10
24
s
(E) 18 x 10
24
s

Gabarito: B
Tipo de questo: Mltipla Escolha
Resolutores: Prof. Me. Dlcio Basso e Prof. Dr. Maria Eullia Pinto Tarrag
Comentrio:
De acordo com a Teoria da Relatividade Restrita (TRR), defini-se tempo
prprio
0
t como o intervalo de tempo medido no prprio referencial no qual o
evento ocorre. Se um observador estiver em outro referencial, deslocando-se com
velocidade constante v em relao ao referencial prprio, o intervalo de tempo
t que ele ir obter para o evento ser dado por
2
2
1
c
v
t
t
o

= ou
o
t t = , sendo
o fator de Lorentz. Como a velocidade v sempre menor do que c, ento sempre
maior do que a unidade (ou igual, para os casos em que v<<c). Isso significa que o
58 Maria Eullia Pinto Tarrag e Dlcio Basso (Orgs.)

ENADE Comentado 2008: Fsica 59
intervalo de tempo medido por qualquer observador que esteja em outro referencial
inercial (que no seja o prprio) ser sempre maior do que o tempo prprio (ou igual,
para os casos em que v<<c); por isso, esse fenmeno chama-se dilatao do
tempo.
Quanto energia, a TRR nos diz que a energia total E de uma partcula com
energia cintica K dada por K E E + =
0
, sendo
2
0 0
c m E = a energia associada
massa de repouso m
0
. A energia total E tambm pode ser dada por
0
E E = .
No LHC, a razo entre a energia total e a energia de repouso para a partcula
de Higgs, caso ela venha a ser detectada 1 , 6
82
500
0
= = =
GeV
GeV
E
E
, seria . Isso
significaria que a vida mdia dessas partculas, medida no referencial do laboratrio,
ser seis vezes a vida mdia medida no referencial prprio, isto
:
o
t t = = s s
25 25
10 18 10 0 , 3 1 , 6

= .
Assim, a alternativa correta a (B).



QUESTO 37
O urnio natural presente na Terra uma mistura de
238
U (99,3%) e
235
U (0,7%). A
vida mdia do
238
U 4,5 bilhes de anos e a do
235
U 1,0 bilho de anos. Supondo
que, na exploso de uma supernova, esses istopos tenham sido produzidos em
quantidades iguais, h quanto tempo, em anos, deve ter ocorrido essa exploso?
(Considere ln(99,3/0,7) = 5)
(A) 6 mil
(B) 20 milhes
(C) 1 bilho
(D) 6 bilhes
(E) 15 bilhes

Gabarito: D
Tipo de questo: Mltipla Escolha
Resolutores: Prof. Dr. Maria Eullia Pinto Tarrag e Prof. Me. Dlcio Basso
Comentrio:
Para resolver essa questo o estudante deve lembrar que as transformaes
radiativas seguem a lei
1
0
1 1

t
e N N

= , na quais
1
N o nmero de ncleos radiativos
do tipo 1 presentes na amostra no tempo t,
0
1
N nmero de ncleos radiativos do
tipo 1 presentes na amostra no tempo inicial t=0 e
1
a vida-mdia dos ncleos do
tipo 1, ou seja, o tempo que em mdia um ncleo permanece sem desintegrar.
De acordo com as informaes da questo, podemos escrever que o nmero
de ncleos de U
238
em funo do tempo
1
0
1 1

t
e N N

= (Eq.1) e o nmero de U
235

em funo do tempo
2
0
2 2

t
e N N

= (Eq.2). Dividindo-se a (Eq.1) pela (Eq.2) obtm-
-se
2
1
0
2
0
1
2
1

t
t
e N
e N
N
N

= =
t
t
t
e
e
e
)
1 1
(
1 2
2
1

= , pois foi dito que esses ncleos foram


produzidos em quantidades iguais (
0
2
0
1
N N = ). Portanto, t
N
N
)
1 1
( ln
1 2 2
1

= .
60 Maria Eullia Pinto Tarrag e Dlcio Basso (Orgs.)

ENADE Comentado 2008: Fsica 61
Inserindo os valores numricos fornecidos obtemos:
anos
N
N
t
9
9 9
1 2
2
1
10 6
)
10 5 , 4
1
10 1
1
(
5
)
1 1
(
ln

=

.
Assim, a alternativa correta a (D).

62 Maria Eullia Pinto Tarrag e Dlcio Basso (Orgs.)
QUESTO 38
O cu azul devido ao espalhamento da luz solar pelas molculas da atmosfera
distribudas de forma inomognea. Este espalhamento, denominado espalhamento
Rayleigh, tambm importante em propagao de luz em fibras pticas, varia com o
inverso da quarta potncia do comprimento de onda (1/
4
).
Considerando essas informaes, analise as explicaes dos fenmenos
apresentados a seguir.
I - Em propagao de luz em fibras pticas de vidro, o Espalhamento Rayleigh
responsvel por uma atenuao maior da intensidade na transmisso ptica
para comprimentos de onda da luz visvel do que para a radiao infravermelha.
II - A cor avermelhada do pr do sol ocorre porque, ao entardecer, os raios solares
incidem tangencialmente superfcie da Terra e as cores de maior freqncia
no conseguem atravessar toda a extenso da atmosfera.
III - A cor azul do cu ocorre porque a luz solar, ao passar pela atmosfera, sofre um
espalhamento maior para as radiaes de menor comprimento de onda do que
para as de maior comprimento de onda.
Est(o) correta(s) a(s) explicao(es)
(A) I, apenas.
(B) I e II, apenas.
(C) I e III, apenas.
(D) II e III, apenas.
(E) I, II e III.

Gabarito: E
Tipo de questo: Mltipla Escolha
Resolutor: Prof. Dr. Cssio Stein Moura
Comentrio:
Vamos discutir cada uma das explicaes propostas na questo:
I Conforme o enunciado afirma, o espalhamento Rayleigh proporcional a
4
1

. A luz visvel tem um comprimento de onda na faixa que vai de


aproximadamente 400 a 780 nm, enquanto que o infravermelho situa-se entre o

ENADE Comentado 2008: Fsica 63
limite de maior comprimento de onda do visvel, 780 nm, e cerca de 1 mm para o
infravermelho longnquo. Como o comprimento de onda do infravermelho maior
que o do visvel, o fator
4
1


ser pequeno para o infravermelho em comparao
com a poro visvel do espectro eletromagntico. Sendo assim, podemos afirmar
que a luz visvel sofre uma atenuao maior que o infravermelho ao se propagar
atravs de fibras ticas.
II Se considerarmos o planeta Terra e a sua atmosfera como esferas
concntricas, podemos afirmar, do ponto de vista geomtrico, que a camada de ar
percorrida por um raio de luz maior quando ele atinge tangencialmente a superfcie
do que quando a atinge perpendicularmente. Dessa forma, o efeito de espalhamento
Rayleigh ser mais evidente no caso da incidncia tangencial, que o caso do pr
do Sol, do que ao meio-dia quando a luz atinge a superfcie perpendicularmente. Os
tons vermelhos do espectro visvel tm comprimentos de onda maiores (ou
frequncias menores) do que os tons azuis, o que faz com que estes sofram um
espalhamento mais intenso do que aqueles, devido ao fator
4
1

, de forma similar
explicao I acima. Devido ao maior espalhamento dos raios azuis, os tons
alaranjados e vermelhos predominam, dando as cores caractersticas do pr do Sol.
III Se o planeta Terra no possusse atmosfera, a luz no sofreria
espalhamento Rayleigh e o cu pareceria negro, como o caso da Lua. Vemos o
cu terrestre da cor azul, porque a luz ao passar pela atmosfera sofre espalhamento:
os tons vermelhos tendem a espalhar menos e a chegar de forma direta na
superfcie, enquanto que os tons azuis so espalhados. A cor que nossos olhos
identificam justamente dos feixes espalhados na atmosfera, dando-lhe a cor
caracterstica azul.
Assim, a alternativa correta a (E).



QUESTO 39 DISCURSIVA

Uma partcula de massa m desliza sem atrito em um anel de raio R. O anel gira com
velocidade angular constante em torno de um eixo vertical, como mostra a figura
acima. A acelerao da gravidade g.

a) Encontre a lagrangiana do sistema, usando como coordenada generalizada o
ngulo definido na figura.
(valor: 3,0 pontos)
b) Escreva a Equao de Euler-Lagrange desse sistema.
(valor: 3,0 pontos)
c) Quantos pontos de equilbrio (estveis ou instveis) existem para < g/R e para
> g/R ?
(valor: 4,0 pontos)

Tipo de questo: Discursiva
Resolutor: Prof. Dr. Sayonara Salvador Cabral da Costa
Comentrio:
a) A lagrangiana L do sistema conservativo da questo a diferena entre a
energia cintica T da partcula e a energia potencial gravitacional U a ela associada,
ou seja, L = T U. Usando como coordenada generalizada , passamos a escrever
estas duas energias:
A energia cintica T da partcula dada pela soma de duas parcelas: a
primeira, T
1
, resultante do movimento da partcula deslizando sobre o anel
64 Maria Eullia Pinto Tarrag e Dlcio Basso (Orgs.)

ENADE Comentado 2008: Fsica 65
(
2 2
1
mR
2
1
T

= ) e a segunda, T
2,
resultante do movimento do anel em torno do eixo
vertical (
2 2
2
) m(Rsen
2
1
T = );
Para a energia potencial gravitacional, U=mgh, pode-se escolher como
referencial U=0 para =0 e h=0. Dessa forma, U=mgR(1cos).
Portanto, mgR mgRcos ) m(Rsen
2
1
mR
2
1
L
2 2 2 2
+ + =

.
b) A equao de Euler-Lagrange, 0
dt
d
=
|
.
|

\
|



L L
, para o sistema resulta,
ento:
0 mR - ) mgR(-sen . cos . mR
2 2 2
= +

sen ,
que pode ser escrita, convenientemente, como:
sen
|
.
|

\
|
=
R
g
cos
2

.
c) Os pontos ou posies de equilbrio sero aqueles para os quais vale a
condio de 0 =

.
Da equao de Euler-Lagrange, obtida acima, v-se que essa condio
implica:
2
R
g
cos ou 0 sen

= = .
No primeiro caso, as posies de equilbrio so =0 e =.
No segundo caso, se
2
>g/R, sendo cos=g/R
2
, resulta que =cos
-1
(g/R
2
);
Ento, se
2
<g/R, resulta que cos>1, o que impossvel, logo as posies
de equilbrio resultam as do primeiro caso, ou seja, =0 (equilbrio estvel) e =
(equilbrio instvel).

66 Maria Eullia Pinto Tarrag e Dlcio Basso (Orgs.)
QUESTO 40 - DISCURSIVA
A figura abaixo mostra o espectro de absoro de vibrao-rotao de uma molcula
diatmica heteronuclear na temperatura ambiente. Para molculas desse tipo, as
energias vibracionais-rotacionais so dadas por

onde
j o nmero quntico rotacional e n o nmero quntico vibracional;
I o momento de inrcia da molcula e
0
a freqncia de vibrao clssica da
molcula. As transies mostradas correspondem s transies com n
final
=1 e
n
inicial
=0 (n = 1) e (j = 1).
(Dados: h = 4,0 x 10
15
eVs;
2
= 10; massa reduzida da molcula = 1,5 x 10
27
kg; (0,360)
2
= 0,13)


a) Apresente a expresso da energia das radiaes absorvidas em funo do
momento de inrcia.
(valor: 4,0 pontos)
b) Calcule o valor numrico aproximado do momento de inrcia da molcula.
(valor: 3,0 pontos)
c) Calcule o valor numrico aproximado da constante elstica da molcula.
(valor: 3,0 pontos)


ENADE Comentado 2008: Fsica 67
Tipo de questo: Discursiva
Resolutor: Prof. Dr. Cssio Stein Moura
Comentrio:
a) Como a molcula est na temperatura ambiente, podemos considerar que
as energias dos estados vibratrios so suficientemente elevadas em comparao
com a energia trmica kT e, portanto, a molcula se encontra no estado vibracional
fundamental no qual 0 = n .
Nessa situao, as regras de seleo permitem que, para a transio
0 =
inicial
n para 1 =
final
n , o nmero quntico rotacional sofra uma variao j = 1, ou
seja, 1 + =
inicial final
j j ou 1 =
final inicial
j j .
A energia dos autoestados vibro-rotacionais dada, conforme o enunciado da
questo, por:

( ) ( ) 1
8 2
1
2
2
0
+ j j
l
h
+ h + n = j n, E
|
.
|

\
|
[1]

Da equao [1] podemos escrever a energia absorvida para cada uma das
transies permitidas:

( ) ( ) ( ) ( )( ) ( )
(

|
.
|

\
|

|
.
|

\
|
1
8 2
1
0 1 1 1
8 2
1
1 0, 1 1, 1 1 0
2
2
0
2
2
0
+ j j
l
h
+ h + + + j + j
l
h
+ h + = j E + j E = + j j , E
( ) ( ) 1
4
1 1 0
2
2
0
+ j
l
h
+ h = + j j , E [2]


( ) ( ) ( ) ( )( ) ( )
(

|
.
|

\
|

|
.
|

\
|
1
8 2
1
0 1 1 1
8 2
1
1 0, 1 1, 1 1 0
2
2
0
2
2
0
+ j j
l
h
+ h + + j j
l
h
+ h + = j E j E = j j , E

( ) j
l
h
h = j j , E
2
2
0
4
1 1 0 [3]

68 Maria Eullia Pinto Tarrag e Dlcio Basso (Orgs.)
b) Analisando o espectro apresentado no enunciado da questo, podemos
extrair os valores das energias de transies a seguir:

E(0 1, 0 1) = 0,363 eV [4]
E(0 1, 1 0) = 0,358 eV [5]

Substituindo [4] e [5] e os respectivos valores de n e j nas equaes [2] e [3],
ficamos com:
| | | |
| | ( )
( ) 1 0
10 4
10 4,0
10 4,0 0,363
2
15
0
15
+
l
s eV
+ s eV = eV



[6]
| | | |
| | ( )
( ) 1
10 4
10 4,0
10 4,0 0,358
2
15
0
15

l
s eV
s eV = eV [7]

Resolvendo [6] e [7] para l encontramos:

2 28
10 1,6 s eV = l

[8]

c) Para os nveis vibracionais mais baixos, a vibrao da molcula diatmica
pode ser comparada a um oscilador harmnico simples, cuja frequncia de oscilao
dada pela relao entre a constante elstica k e a massa efetiva :

=
2
1
0
[9]

Para calcular a constante elstica do oscilador usando a equao [9],
precisamos conhecer o valor de sua frequncia fundamental. Para encontr-la,
substitumos o valor de l definido em [8] na equao [6] (ou na [7]):

| | | |
| | ( )
| |
2 28
2
15
0
15
10 1,6 10 4
10 4,0
10 4,0 0,363
s eV
s eV
+ s eV = eV



[10]

Com isso, chegamos a

ENADE Comentado 2008: Fsica 69
Hz =
13
0
10 9,0 [11]

Substituindo [11] em [9],

27
13
10 1,5 2
1
10 9,0

= [12]

e, resolvendo para k, temos que:

m N = k / 10 4,8
2
.

























COMPONENTE ESPECFICO
LICENCIATURA

ENADE Comentado 2008: Fsica 71
QUESTO 41
Para avaliar se os estudantes haviam superado concepes comuns s da teoria
medieval do impetus em relao compreenso dinmica da situao estudada, o
professor props o problema apresentado a seguir.


Qual das seguintes seria a resposta tpica de um aluno dito newtoniano?
(A) A fora com que a bola foi lanada diminui com o tempo, at se igualar, na
posio de altura mxima, soma das foras peso e atrito com o ar.
(B) A fora com que a bola foi lanada diminui pela ao do atrito com o ar, at se
igualar ao peso da bola na posio de altura mxima.
(C) As foras que agem sobre a bola aps o lanamento agem no sentido contrrio
ao movimento na subida, e a favor do movimento, na descida.
(D) As foras que agem sobre a bola aps o lanamento agem no sentido contrrio
ao movimento na subida, e em ambos os sentidos, na descida.
(E) As foras que agem sobre a bola aps o lanamento agem no mesmo sentido
que o movimento na subida e na descida.

Gabarito: D
Tipo de questo: Mltipla Escolha
Resolutor: Prof. Dr. Joo Bernardes da Rocha Filho
Comentrio:
Essa uma questo que tenta avaliar se a noo de fora versus movimento
do estudante universitrio aristotlica ou newtoniana, e se ele compreende
adequadamente as ideias de atrito e peso.
A questo pode ser resolvida pensando-se que o movimento de uma bola
lanada nessas condies pode ser subdividido em trs momentos distintos, para
efeitos de anlise:
1) o instante do lanamento, isto , o curto intervalo de tempo no qual o p do
lanador, ou outro mecanismo de lanamento, encosta na bola;
Uma bola de futebol lanada verticalmente para cima, a partir do telhado de
um edifcio de altura h
0
, com velocidade v
0
. Apresente uma explicao relativa
ao lanamento, que leve em conta a resistncia do ar.

72 Maria Eullia Pinto Tarrag e Dlcio Basso (Orgs.)
2) o perodo que vai do instante em que o lanador desencosta da bola at
que ela atinja a altura mxima, e;
3) o perodo que corresponde a descida da bola.

Sobre esses trs momentos podemos afirmar que:
Tanto o lanador quanto a bola so entidades em certa medida elsticas, a
fora com que a bola lanada varia ao longo do intervalo de tempo em que o
lanador fica em contato com ela, mas cessa imediatamente quando termina o
contato da bola com o lanador, pois justamente esse contato o meio atravs do
qual a fora aplicada. No havendo contato, no h fora de lanamento, por isso
ela termina quando a bola se liberta do contato com o lanador.
As foras que agem sobre a bola aps o lanamento so (basicamente) duas:
o atrito com o ar e o peso da bola. O peso age puxando a bola para baixo durante
toda a trajetria (subida e descida), j que depende apenas da ao gravitacional, e
no do movimento da bola. O atrito, por sua vez, uma fora que se ope ao
movimento do objeto, portanto, o atrito da bola com o ar uma fora que aponta
para baixo, durante sua subida e, para cima, durante sua descida.
Com base nas consideraes acima, podemos agora avaliar cada uma das
alternativas oferecidas na questo:
(A) est errada, pois a fora com que a bola foi lanada se reduz a zero
imediatamente na separao da bola e do lanador;
(B) est errada, pela mesma razo do item (A);
(C) est errada, pois as foras que atuam na descida da bola so o peso, que
atua a favor do movimento, e o atrito, que atua contra o movimento;
(D) est correta, pois na subida a fora peso e a fora de atrito com o ar se
opem ao movimento, mas na descida o peso favorece o movimento
enquanto o atrito se ope a ele, apontando em sentidos opostos;
(E) est errada, pois na subida o atrito e o peso se opem ao movimento, e
na descida o peso age no mesmo sentido do movimento, enquanto o
atrito se ope a este.

ENADE Comentado 2008: Fsica 73
QUESTO 42
Hertz, no experimento em que evidenciou a existncia das ondas eletromagnticas,
notou que a descarga eltrica no sensor era mais facilmente percebida quando este
era iluminado com luz de freqncia acima de um certo valor. A explicao de
Einstein para este efeito, denominado fotoeltrico, considera que
(A) o aumento da intensidade da luz implica um aumento do nmero de ftons de
mesma energia que incide sobre o sensor.
(B) o intervalo de tempo entre a chegada da luz ao sensor e a emisso dos
eltrons diferente de zero.
(C) a luz se comporta como onda no momento em que ocorre o efeito.
(D) a energia dos eltrons que saem do sensor depende diretamente da
intensidade de luz incidente.
(E) a energia do fton incidente igual energia cintica do eltron atingido.

Gabarito: A
Tipo de questo: Mltipla Escolha
Resolutor: Prof. Dr. Joo Bernardes da Rocha Filho
Comentrio:
O objetivo dessa questo avaliar se o estudante compreende o efeito
fotoeltrico.
O texto da questo alude a um experimento realizado por Hertz, e se refere
explicitamente relao entre a frequncia da radiao luminosa emitida e a
ocorrncia perceptvel de descargas eltricas entre sensores, associando esse
fenmeno ao efeito fotoeltrico estudado por Einstein. O cerne do efeito fotoeltrico
envolve justamente o compromisso entre a frequncia da radiao emitida e a
capacidade desta em produzir liberaes de eltrons em superfcies metlicas.
Para responder questo o estudante deveria saber que:
1) a intensidade de uma fonte luminosa est relacionada quantidade de
ftons que ela emite em certo tempo, ou seja, a taxa de emisso de
ftons;
2) o efeito fotoeltrico um fenmeno que sugere que a radiao
luminosa, normalmente considerada como onda eletromagntica, tem
tambm comportamento de partcula;

74 Maria Eullia Pinto Tarrag e Dlcio Basso (Orgs.)
3) a energia com que os eltrons so liberados de uma superfcie metlica
iluminada funo da energia da radiao incidente e da funo
trabalho do metal;
4) a intensidade da radiao incidente tem relao com a quantidade de
eltrons liberados em certo intervalo de tempo, ou seja, que a taxa de
ftons incidentes tem relao com a taxa de liberao de eltrons;
5) parte da energia do fton incidente absorvida para garantir a liberao
do eltron, e que a energia excedente do fton transformada em
energia cintica do eltron;
6) o efeito fotoeltrico um fenmeno quntico para o qual no est
definido um intervalo de tempo e, portanto, podemos dizer que o eltron
liberado simultaneamente incidncia do fton.
Com base nas informaes acima, podemos agora avaliar cada uma das
alternativas oferecidas nessa questo:
(A) est correta, pois o aumento na intensidade da luz precisamente o
aumento na taxa com que os ftons so liberados pela fonte luminosa e,
portanto, na taxa com que atingem o sensor;
(B) est errada, pois considera-se que o efeito fotoeltrico, por ser quntico,
no tem tempo definido para sua ocorrncia, sendo instantneo;
(C) est errada, pois o efeito fotoeltrico justamente um fenmeno que
envolve a expulso de eltrons superficiais de metais, o que sugere que
os ftons tm momento linear e, portanto, comportam-se como partculas;
(D) est errada, pois a quantidade dos eltrons liberados pela radiao est
relacionada com a intensidade da radiao luminosa incidente, enquanto
a energia desses eltrons tem relao com a frequncia da luz incidente e
com a funo trabalho do metal;
(E) est errada, pois uma parte da energia do fton incidente utilizada para
extrair o eltron de sua rbita atmica, e somente a parte restante
transformada em energia cintica.



QUESTO 43
H uma variedade de possibilidades e tendncias do uso de estratgias de ensino
frutferas para se ensinar de modo significativo e consistente. Uma abordagem
construtivista a ser adotada no laboratrio didtico a que apresenta situaes
experimentais destinadas a que os alunos
(A) verifiquem e confirmem leis e teorias da Fsica, previamente ensinadas.
(B) revelem qualitativamente suas idias prvias.
(C) exemplifiquem o uso da metodologia cientfica na produo da cincia.
(D) redescubram a cincia produzida por cientistas.
(E) evitem o desenvolvimento de concepes alternativas cientfica.

Gabarito: B
Tipo de questo: Mltipla Escolha
Resolutor: Prof. Dr. Joo Bernardes da Rocha Filho
Comentrio:
O objetivo da questo avaliar o conhecimento que os estudantes tm acerca
do construtivismo, e para isso apresenta uma situao hipottica envolvendo um
experimento proposto por um professor em um laboratrio didtico, procurando
identificar, entre as alternativas, a que representa melhor uma abordagem
construtivista adequada para a situao experimental.
Para responder essa questo o estudante deveria saber que o construtivismo
uma epistemologia com aplicao metodolgica e didtica no ensino das cincias,
que considera que o conhecimento no algo pronto, a ser adquirido, assimilado ou
transmitido, mas sim elaborado internamente em cada indivduo pela interao que
realiza com o meio, com a produo cientfica da humanidade, com a cultura popular
e com as demais pessoas. Nesse sentido, o conhecimento produto da ao, que
por sua vez fundada nos conhecimentos prvios que todos tm, e que, portanto,
precisam ser considerados no processo educativo.
Com base nas informaes acima, podemos agora avaliar cada uma das
alternativas oferecidas na questo:

ENADE Comentado 2008: Fsica 75

76 Maria Eullia Pinto Tarrag e Dlcio Basso (Orgs.)
(A) est errada, pois a verificao e a confirmao de leis e teorias da Fsica,
previamente ensinadas, caracteriza um ensino tradicional, centrado no
conhecimento como algo externo, pronto, dominado pelo professor, a ser
assimilado pelo educando;
(B) est correta, pois um dos objetivos do uso do laboratrio ou da
experimentao no contexto construtivista fazer aflorar os pr-
conhecimentos dos estudantes, permitindo que eles os manifestem por
meio da manipulao da realidade. A partir desses pr-conhecimentos e
de conhecimentos obtidos de todas as outras fontes acessveis, , ento,
possvel que o estudante empenhe-se em um processo de reconstruo
prpria do conhecimento cientfico;
(C) est errada, pois a metodologia cientfica baseada no pressuposto de
que o conhecimento pode ser atingido pela utilizao de mtodos formais,
hipottico-dedutivos e lineares, que o construtivismo no reconhece como
teis no contexto didtico de um laboratrio de Fsica, pois eles exigem
pr-conhecimentos homogneos, incluindo metodologias padro, que no
se originam no prprio indivduo;
(D) est errada, pois a redescoberta da cincia produzida por cientistas
pressupe que o caminho que leva ao conhecimento educacional o
mesmo que leva s descobertas cientficas, como relatadas nos livros de
Histria da Cincia. Para o construtivismo, alm da repetio ser
naturalmente contraproducente em termos de aprendizagem, o caminho
seguido pelos cientistas para a descoberta de leis frequentemente
intuitivo e serendpico, e dificilmente pode ser reproduzido em laboratrios
didticos em funo do tempo, da dedicao extrema que demandam, e
dos condicionantes histricos diferentes;
(E) est errada, pois as concepes alternativas existem naturalmente nos
estudantes, e a ida ao laboratrio de Fsica, no contexto construtivista,
objetiva exatamente que essas concepes aflorem e sejam questionadas
pelos prprios estudantes, a partir dos resultados que encontram na
manipulao da realidade e nas demais fontes de consulta que puderem
acessar.

ENADE Comentado 2008: Fsica 77
QUESTO 44
Os Parmetros Curriculares Nacionais (PCN) redirecionaram a Fsica, no Ensino
Mdio, para estimular, nos alunos, o interesse por conhecer o mundo fsico a partir
de procedimentos para formar cidados autnomos intelectualmente. Considerando
esse referencial, analise as seguintes abordagens presentes em materiais didticos:

I - procedimentos de pesquisa de concepes de senso comum;
II - privilgio de aspectos tericos;
III - utilizao de novo saber em sua dimenso aplicada;
IV - apresentao do conhecimento como fruto da genialidade dos cientistas.

Para selecionar materiais didticos que atendam s orientaes dos PCN para o
Ensino Mdio, devem ser consideradas APENAS as abordagens
(A) I e III
(B) I e IV
(C) II e III
(D) II e IV
(E) III e IV

Gabarito: A
Tipo de questo: Mltipla Escolha
Resolutor: Prof. Dr. Joo Bernardes da Rocha Filho
Comentrio:
O objetivo da questo avaliar nos estudantes a capacidade de interpretar
uma diretriz dos PCNs, confrontando-a com possveis abordagens em um trabalho
pedaggico, verificando quais delas so compatveis com aquela diretriz.
Para responder essa pergunta necessrio saber que um dos objetivos do
ensino de Fsica formar cidados autnomos intelectualmente. Essa autonomia
intelectual se manifesta, entre outras formas, pela capacidade de distinguir os
conhecimentos do senso comum dos conhecimentos cientficos, pois assim os
cidados sero capazes de fazer escolhas adequadas, considerando um mesmo
problema sob diversos aspectos e sabendo, ainda, argumentar a favor ou contra
procedimentos ou posicionamentos, segundo seus pressupostos. Alm disso, os

78 Maria Eullia Pinto Tarrag e Dlcio Basso (Orgs.)
cidados intelectualmente autnomos devem ser capazes de avaliar as implicaes
da utilizao do conhecimento cientfico, que surge na forma de produtos e processos
potencialmente danosos para o meio ambiente, por exemplo, evitando a manipulao
que o poder econmico pode exercer sobre a populao via meios de comunicao.
Assim, podemos dizer que o:
1. O item I cita uma abordagem til ao alcance da diretriz dos PCNs citada no
enunciado da questo, pois com base nela o estudante estimulado a
instrumentalizar-se para compreender a cincia e o pensamento cientfico,
confrontando-o com o senso comum, o que o auxilia a atingir a autonomia
intelectual desejada;
2. O item II til para a formao de cientistas, porm tem pouca relao com a
formao de cidados autnomos ou com o estmulo para que os alunos
conheam o mundo fsico. Ao contrrio, frequente que a nfase nos
aspectos tericos do conhecimento fsico seja compreendida pelos estudantes
do nvel mdio como uma barreira aprendizagem da Fsica, o que os afasta
da cincia e entrava o desenvolvimento da autonomia que se deseja.
3. O item III se relaciona com a contextualizao no ensino de Fsica, que
um recurso pedaggico que contribui para formar cidados capazes de
avaliar a pertinncia, a utilidade e a nocividade de novos produtos e
tecnologias, levando-os, posteriormente, ao desenvolvimento da
capacidade de tomar partido, por exemplo, quanto pertinncia ou no do
financiamento pblico de pesquisas cientficas para um dado tema, que
um dos papis esperados de um cidado completo.
4. O item IV um equvoco baseado em um erro epistemolgico porque mais
afasta os estudantes da cincia do que os aproxima dela, mostrando uma
cincia irreal, feita por gnios excntricos. Nem isso verdade, j que os
cientistas so pessoas comuns, que tm famlias e vivem em sociedade,
nem essa estria romanceada e permeada de gnios da cincia ajuda a
despertar novas vocaes cientficas, pois produz um rebaixamento da
autoestima dos estudantes.
Sendo assim, as afirmaes corretas so a I e a III, portanto a alternativa
correta a letra (A).

ENADE Comentado 2008: Fsica 79
QUESTO 45
Calor e temperatura so conceitos estatsticos ligados s propriedades coletivas das
partculas que constituem os corpos: a temperatura est ligada energia cintica
mdia das partculas e o calor, s trocas de energia entre os constituintes dos
corpos. Ao utilizar em aula um termoscpio, o professor, associando discusses
histricas ao experimento, possibilitar que seus alunos distingam os conceitos de
temperatura e calor, ao constatarem que, quando ele segura o termoscpio, o nvel
do lquido

(A) aumenta, caso a temperatura do professor seja superior do ambiente.
(B) aumenta, caso a temperatura do professor seja igual do ambiente.
(C) aumenta, para qualquer temperatura ambiente.
(D) no se altera, caso a temperatura do professor seja menor que a do ambiente.
(E) diminui, caso a temperatura do professor seja maior que a do ambiente.

Gabarito: A
Tipo de questo: Mltipla Escolha
Resolutor: Prof. Dr. Joo Bernardes da Rocha Filho
Comentrio:
Essa questo objetiva verificar se o estudante compreende o que um
termoscpio e se capaz de utilizar a lei do equilbrio termodinmico em uma
situao experimental, alm de ter conhecimento sobre a dilatao trmica. O
termoscpio uma espcie de termmetro de lquido em vidro podendo conter dois
bulbos e um tubo, ou um bulbo e um tubo, contendo lquido e ar em seu interior.
Existem muitas variaes modernas do termoscpio, porm vamos fixar-nos, nessa
anlise, no termoscpio galileano, que era aberto na parte superior e usava gua, e
no termoscpio de Mdici, que era fechado na parte superior, e usava lcool.
Vamos analisar, uma a uma, as alternativas, propostas a partir da afirmao
de que o professor segura o termoscpio, assumindo que o termoscpio esteja em
equilbrio termodinmico com o ambiente antes do professor segur-lo:


80 Maria Eullia Pinto Tarrag e Dlcio Basso (Orgs.)
(A) essa alternativa prope que o nvel do lquido no instrumento aumente
caso a temperatura do professor seja maior do que a do ambiente. Essa
uma alternativa correta, pois o lquido e o ar do bulbo inferior,
inicialmente em temperatura ambiente, sero aquecidos e dilataro mais
que o vidro do recipiente. Com isso, a presso no interior do bulbo inferior
aumentar em relao presso atmosfrica ou presso existente no
bulbo superior, ainda frio, empurrando para cima o lquido pelo tubo
vertical (aberto ou fechado, conforme o tipo de termoscpio);
(B) essa alternativa est errada, pois se no h diferena de temperatura
entre a mo do professor e dos materiais do termoscpio, no haver
variao da presso interna, e nenhum lquido ser empurrado atravs do
tubo vertical;
(C) essa alternativa est errada, pois a temperatura ambiente influencia
diretamente o funcionamento do termoscpio, como vimos acima;
(D) essa alternativa est errada, pois o lquido deveria baixar no tubo vertical,
j que a presso interna do termoscpio iria diminuir com o resfriamento
causado pela mo do professor;
(E) essa alternativa est errada, pois precisamente o oposto do que foi
afirmado no item (A).

ENADE Comentado 2008: Fsica 81
QUESTO 46 (QUESTO ANULADA)
Com relao produo e utilizao das Tecnologias da Informao e da
Comunicao (TIC), pode-se afirmar que:

I - a TIC deve adaptar-se s necessidades de determinado projeto poltico-
pedaggico;
II - a introduo de novas TIC na educao implica novas prticas pedaggicas;
III - o planejamento das TIC deve permitir a reflexo dos estudantes sobre diferentes
abordagens de um mesmo problema.
Est(o) correta(s) a(s) afirmao(es)
(A) I, apenas.
(B) II, apenas.
(C) I e III, apenas.
(D) II e III, apenas.
(E) I, II e III.

Gabarito: C
Tipo de questo: Mltipla Escolha
Resolutor: Prof. Dr. Aldoir Rigoni
Comentrio:
Essa questo trata da produo e uso das Tecnologias da Informao e
Comunicao (TIC) nos processos educacionais e de formao profissional nas
Instituies de Ensino. As novas tecnologias educacionais esto presentes nas
Instituies de Ensino em maior ou menor intensidade e podemos dizer que so
imprescindveis. As TIC so itens de verificao das Comisses de Especialistas do
MEC, tanto na avaliao para o reconhecimento de cursos como nas verificaes
das condies de ensino. O propsito da questo o de verificar em que grau e
como o estudante percebe e acompanha a presena desses novos recursos
tecnolgicos na ao didtico-pedaggica. Recursos estes constantes nos Projeto
Pedaggico do Curso PPC.


82 Maria Eullia Pinto Tarrag e Dlcio Basso (Orgs.)
Em relao s alternativas de resposta questo formulada podem ser feitas
as seguintes consideraes:
a) a alternativa I est correta, pois a Tecnologia da Informao e
Comunicao deve estar em consonncia com as necessidades previstas no Projeto
Pedaggico do Curso PPC, que na sua construo prev os recursos tecnolgicos
necessrios ao desenvolvimento do ensino.
b) A alternativa II no procedente, pois as novas Tecnologias esto a
servio das prticas pedaggicas e no necessariamente determinam novas
prticas.
c) A alternativa III est correta, pois um mesmo problema sempre deve
permitir variadas abordagens, o que feito, via de regra, com o uso de diferentes
tecnologias de ensino, facilitando a reflexo dos estudantes sobre o mesmo assunto
com o envolvimento de uma quantidade maior de faculdades mentais no processo.
Assim a resposta correta a letra (C).

ENADE Comentado 2008: Fsica 83
QUESTO 47
O desenvolvimento da ao educativa a partir da construo de projetos poltico-
-pedaggicos tornou-se uma obrigao para as escolas. Neste contexto, a Fsica
deve participar, aproveitando os momentos pedaggicos para trabalhar seus
contedos. Numa escola hipottica, a realizao de uma pea teatral um desses
momentos. Os professores de Fsica resolvem trabalhar, assim, com o tema
estruturador 4 dos PCN+: Som, Imagem e Informao. O projeto da Fsica trabalhar
com a iluminao do palco, sem esquecer as diretrizes do cengrafo. As
caractersticas do teatro so: as paredes do palco, quando iluminadas com luz verde
e vermelho, misturadas, ficam amarelas, e o piso, quando iluminado com luz verde e
azul, misturadas, fica ciano. As cores das paredes do palco e do piso,
respectivamente, so:
(A) amarelo e azul.
(B) amarelo e amarelo.
(C) verde e magenta.
(D) azul e branco.
(E) branco e branco.

Gabarito: E
Tipo de questo: Mltipla Escolha
Resolutor: Prof. Dr. Aldoir Rigoni
Comentrio:
Na resoluo dessa questo o estudante deve saber que a luz branca forma-
-se da composio de todas as cores do espectro. Uma superfcie branca tem a
propriedade de refletir todos os raios luminosos que nela incidem.
A cor amarela uma cor secundria resultante da sobreposio das cores
vermelha e verde. A incidncia das cores vermelho e verde em paredes brancas as
tornam amarelas, pois esta a composio resultante da mistura. Assim, as paredes
do palco devem ser brancas.
O mesmo raciocnio pode ser feito em relao cor ciano, resultante da
mistura das luzes azul e verde, portanto o palco tambm deve ter cor branca. Assim,
considerando-se o que foi exposto, conclui-se que a alternativa correta a letra (E).

84 Maria Eullia Pinto Tarrag e Dlcio Basso (Orgs.)
QUESTO 48
No vocabulrio pedaggico do MEC, presente nos Parmetros Curriculares
Nacionais (PCN), interdisciplinaridade, contextualizao e autonomia so trs pilares
fundamentais da Educao. Nessa perspectiva, procurando seguir as orientaes
oficiais dos PCN, os currculos escolares apresentam algumas das recomendaes
abaixo.

I - A interdisciplinaridade no deve preceder a disciplinaridade.
II - Uma referncia fundamental considerar o que o jovem precisa para viver em
um mundo tecnolgico complexo e em transformao.
III - As disciplinas afins devem ser agrupadas em uma nica disciplina.
IV - A lista de tpicos dos programas no deve ser o foco principal.
Esto de acordo com as orientaes oficiais APENAS os currculos que seguem as
recomendaes
(A) I e II
(B) I, II e III
(C) I, II e IV
(D) I, III e IV
(E) II, III e IV

Gabarito: Questo C
Tipo de questo: Mltipla Escolha
Resolutor: Prof. Dr. Aldoir Rigoni
Comentrio:
Autonomia, contextualizao e interdisciplinaridade formam o trip de uma
nova maneira de ver a educao adaptada aos nossos tempos.
A organizao e tratamento dos contedos devem contemplar a
interdisciplinaridade que garante o inter-relacionamento das disciplinas em reas
especficas. notrio que a interdisciplinaridade posterior definio das
disciplinas, pois aquela no subsiste sem estas. A interdisciplinaridade permite o
aperfeioamento das estruturas curriculares nas escolas.


ENADE Comentado 2008: Fsica 85
A contextualizao relaciona de maneira perceptvel o contedo ensinado nas
escolas com os mbitos ou dimenses presentes na vida pessoal, social e cultural
dos alunos. Incentiva comportamento ativo e participante na busca do que precisam
para viver num mundo competitivo, tecnolgico e de permanente transformao. Por
falta de contextualizao muitas pessoas que estudaram fsica na escola talvez no
consigam entender como funciona um celular, um controle eletrnico ou no saibam
estabelecer relaes entre o aquecimento de gua e o consumo de energia eltrica,
ou de gs.
No necessariamente as disciplinas afins devem ser reunidas em uma nica,
constituindo um eixo ou bloco de ensino. Essa ao pode acarretar dificuldades de
ordem acadmico-administrativas, relacionadas aos crditos e ao grande
entrosamento pedaggico que deveria ocorrer entre os professores que fossem
ministrar o conjunto de disciplinas reunidas.
De outra parte procede afirmar que a lista de tpicos dos programas no deve
ser o principal, uma vez que a escolha dos tpicos necessariamente precedida
pelos objetivos e pelo perfil profissional definidos nos Projetos Pedaggicos dos
Cursos.
A partir das consideraes feitas possvel concluir que somente o item III
no est correto, estando todas as demais recomendaes de acordo com as
orientaes oficiais. Logo, a alternativa (C) a correta.

86 Maria Eullia Pinto Tarrag e Dlcio Basso (Orgs.)
QUESTO 49 - DISCURSIVA
Reconhecendo que os sistemas democrticos se tornam vulnerveis sem a cultura
cientfica, um professor de Fsica concordou com as preocupaes expressas nos
Parmetros Curriculares Nacionais (PCN) sobre a formao do cidado e com as
sugestes de mudanas curriculares a serem adotadas nas escolas.
Nessa perspectiva, levando em conta os aspectos contextualizadores no cotidiano e
na Histria da Cincia e os aspectos epistemolgicos e metodolgicos do Ensino da
Fsica, descreva uma atividade a ser realizada em uma Unidade de Ensino sendo o
tema estruturador Universo, Terra e Vida, para cada habilidade abaixo.

a) Adquirir uma compreenso atualizada das hipteses, modelos e formas de
investigao sobre a origem e evoluo do Universo.
(valor: 5,0 pontos)

b) Identificar formas pelas quais os modelos explicativos do Universo influenciaram
a cultura e a vida humana ao longo da histria da humanidade e vice-versa.
(valor: 5,0 pontos)

Tipo de questo: Discursiva
Resolutores: Prof. Dr. Maria Eullia Pinto Tarrag e Prof. Me. Dlcio Basso
Comentrio:
a) Atividades propostas para desenvolver a compreenso atualizada das
hipteses, modelos e formas de investigao sobre a origem e evoluo do
Universo:
Leitura de captulos de livros, visita a stios na internet e projeo de filmes
seguidos de discusso que tratem sobre o tema, como, por exemplo:
Livros:
Descobrindo o Universo, Neil F. Comins & William J. Kaufmann III, Ed.
Bookmann. 2010
Guia Ilustrado Zahar: Astronomia. Ian Ridpath. Jorge Zahar Editor.
Filmes:
De olho no Cu, produzido e divulgado pela IAU, no Ano Internacional da
Astronomia (2009).

ENADE Comentado 2008: Fsica 87
Srie Cosmos. Carl Sagan. Editora Abril.
The Universe. The History Channel.

Stios na internet (sites):
http://www.telescopiosastronomicos.com.br/
http://www.pucrs.br/fisica/astronomia
http://cdcc.sc.usp.br/cda/aprendendo-basico/universo/universo.htm

Ao final dessas atividades o aluno dever elaborar um texto que, na sua
opinio, destaque o que lhe pareceu mais significativo.

b) Formas como os modelos explicativos do universo influenciaram a vida humana e
a cultura ao longo da histria da humanidade.
Ao longo do tempo, a observao do Universo, quer a olho nu ou com
telescpio, acabou levando a mudanas nas concepes cosmolgicas e nas fsicas
que lhe do suporte. Por exemplo:
A cosmologia e a fsica aristotlicas acabaram sendo substitudas por uma
cosmologia heliocntrica. Essa nova concepo cosmolgica de colocar o Sol no
centro e a Terra como planeta gerou uma nova fsica, com as Leis do Movimento e a
Lei da Gravitao, a qual serviu de ponto de partida para o desenvolvimento de
outras reas da fsica como a termodinmica e o eletromagnetismo. Estas, por sua
vez, levaram ao aperfeioamento das mquinas trmicas que propiciaram a
Revoluo Industrial e ao surgimento da era da Eletricidade, que levou at a era das
telecomunicaes e da telemtica, era que estamos vivendo atualmente.

88 Maria Eullia Pinto Tarrag e Dlcio Basso (Orgs.)
QUESTO 50 - DISCURSIVA
Nos circuitos das Figuras 1 e 2 abaixo, as pilhas e as lmpadas so idnticas. Ao
prever o brilho da lmpada L1 em relao aos brilhos das lmpadas L2 e L3, nos
dois circuitos, muito comum que alunos do Ensino Mdio apresentem concepes
alternativas s concepes cientficas.



a) A esse respeito, apresente uma concepo cientfica e uma possvel concepo
alternativa, com a justificativa que os alunos poderiam apresentar.
(valor: 5,0 pontos)

b) Descreva uma estratgia de ensino contextualizada para que os alunos avancem
em direo ao conhecimento cientfico, realizando aprendizagem significativa dos
conceitos de corrente eltrica, resistncia eltrica, resistncia equivalente e
diferena de potencial. Indique nessa estratgia como o mundo vivencial dos
alunos e as relaes de Cincia, Tecnologia e Sociedade (CT&S) podem ser
considerados e os recursos metodolgicos a serem utilizados.
(valor: 5,0 pontos)

Tipo de questo: Discursiva
Resolutores: Adriana Schier e Prof. Dr. Sayonara Cabral da Costa
Comentrio:
Concepo cientfica: inicialmente as trs lmpadas esto submetidas
mesma diferena de potencial V do gerador, pois no 2circuito L2 e L3 esto ligados
em paralelo com o gerador e no 1circuito L1 a nica lmpada. Ento observando

ENADE Comentado 2008: Fsica 89
os desenhos, no ocorre mudana no brilho das lmpadas, mesmo se uma ligao
em srie e a outra em paralelo.
Concepo alternativa: uma grande parte dos alunos acredita que na figura 2
as lmpadas teriam um brilho menor, como se a energia fosse dividida por dois[...]
Apesar de brilharem igualmente entre si, L1 teria um brilho maior do que o de L2 e
L3. Mas L2 e L3 brilham igualmente.
Como R2 = R3 = R
1/Req = 1/R2 + 1/R3 = 2/R ento Req = R/2

Com essa demonstrao matemtica podemos observar que a resistncia
equivalente ir diminuir e a corrente ira aumentar.
A corrente eltrica na figura 1(a partir de V = iR):
V = i . Req i = V/R
Corrente eltrica na figura 2:
i = V i = 2V/R
R/2
A energia do gerador P = Ri
2
, os alunos no se do conta de que a corrente
na figura 1 a mesma da figura 2. Em uma associao em paralelo, a corrente
aumenta, a resistncia diminui e o brilho das lmpadas igual.
Resposta para item (b):
Para comear, a estratgia de ensino que pode ser utilizada, a visualizao
(atividade experimental) das associaes (mostrando srie e paralelo), com fio de
ligao, pilhas e lmpadas. No explicando de incio o fenmeno fsico que ocorre,
logo ir despertar o interesse no aluno, porque a prpria demonstrao certifica que
sua concepo est um pouco equivocada, e a reao do aluno de curiosidade.
Com isso, prendemos sua ateno para maiores explicaes, que ser o passo
seguinte.


90 Maria Eullia Pinto Tarrag e Dlcio Basso (Orgs.)


Na parte vivencial, a instalao eltrica de nossas casas feita por
associao em paralelo. Nesse caso, possvel que todas as tomadas e ponto de
luz tenham a mesma diferena de potencial. O que vai mudar a corrente que vai
circular em cada lmpada, por cada dispositivo eltrico, a resistncia das lmpadas
diferente e depende da sua potncia, ento quanto mais lmpadas uma residncia
possui, maior ser a corrente eltrica que circula pela instalao.
Os clculos matemticos provm de equaes fsicas simples, que podem ser
facilmente compreendidas.

ENADE Comentado 2008: Fsica 91
LISTA DE CONTRIBUINTES
Adriana Schier
Adriano Moehlecke
Aldoir Rigoni
Alexandre Ferret
Aline Cristiane Pan
Artur Majolo Scheid
Cssio Stein Moura
Dlcio Basso
Elaine Evani Streck
Elias Cantarelli Hoffmann
Izete Zanesco
Janana Galho Borges
Joo Bernardes da Rocha Filho
Juliane Bernardes Marcolino
Maiara Oliveira Dalenogare
Mrcio Galhardi
Maria do Carmo Baptista Lagreca
Maria Eullia Pinto Tarrag
Natthan Ruschel Soares
Rafael L. Zimmer
Ricardo Meurer Papalo
Sayonara Salvador Cabral da Costa

Das könnte Ihnen auch gefallen